MK Ka Paeds Bazooka

Download as pdf or txt
Download as pdf or txt
You are on page 1of 150

MK

PAEDIATRICS ULTIMATE REVISION


SECTION 1: SINGLE BEST ANSWER
1. A 15-year-old girl presents to you with history of fever and abdominal pain for 3
weeks. There is history of constipation for 4 days and a transient rash which has
since disappeared. On examination the patient is ill looking. The temperature is
40o C while the pulse rate is 67/min. She has also presented with a slight cough
with no physical signs. What would be the most likely causative organism?
A. Entero-invasive Escherichia coli
B. Campylobacter jejuni
C. Salmonella typhi
D. Helicobacter pylori

2. The commonest organisms causing meningitis in neonates:


A. Streptococcus pneumonia and Haemophilus influenza
B. Streptococcus pneumonia and Neisseria Meningitidis
C. Group B B-haemolytic streptococci and Escherichia coli
D. Group B streptococci and Listeria monocytogenes

3. A 10-year-old girl is noted to have cardiac murmur. Two days prior to admission,
she complained of fever and headache and suddenly lost consciousness today. On
examination, she is noted to be febrile and has a left sided hemiplegia. The most
likely reason for the development of the hemiplegia is:
A. Thrombo-embolic phenomena secondary to infective endocarditis
B. Hypertensive crisis
C. Tetralogy of Fallot
D. Development of meningitis with complications

4. A 7-year-old girl presents with chorea following a streptococcal throat infection.


In addition, she is noted to have tachycardia, a gallop rhythm and soft mitral
regurgitation murmur. Of the following interventions, chronic disability is MOST
likely to be prevented by
A. Intravenous penicillin for 6 weeks
B. Intramuscular penicillin monthly
C. Oral prednisolone for 4 weeks
D. Oral salicylates for 6 weeks
MK

5. A 3-year-old child presents with history of an abdominal swelling for 2 months


duration. A month ago, the mother noticed the swelling whilst bathing the child.
What is the most likely diagnosis?
A. Wilms tumor if the spleen enlarged on the affected side
B. Neuroblastoma if the mass is supra-pubic
C. Leukemia if the child is pale with petechial lesions
D. Sickle cell anemia if the liver is enlarged

6. The following are true of Infective endocarditis, except:


A. Can be caused by Streptococcus aureus.
B. Hepato-splenomegaly is a characteristic feature
C. Hematuria may be a feature.
D. Treatment with intravenous antibiotics for 6 weeks is recommended.

7. The following laboratory results were returned in a 6-week-old boy admitted with
6 days of severe projectile vomiting:
pH 7.51 (7.35-7.45), PO2 12 KPa (95 mmHg), PCO2 4.7 KPa (35 mmHg), Blood
Urea 11 mmol/l, Na+ 131 mmol/l, K+ 3 mmol/l, Cl- 83 mmol/l
Which of the following is true concerning this patient?
A. He has hypertrophic pyloric stenosis
B. He is likely to have a bulging anterior fontanelle
C. He should be resuscitated immediately with normal saline
D. He should be commenced immediately on half strength soy protein, low
lactose formula

8. A 15-month-old child with flexural eczema presents with a 3-day history of cough
and wheeze. On examination his saturations are 92% in air, with a respiratory
rate of 55/min and moderate recession. The heart rate is 150/min. What is the
most likely diagnosis?
A. Gastroesophageal reflux
B. Allergic rhinitis
C. Bronchiolitis
D. Asthma
MK

9. Which of the following is true of tetanus?


A. Failure to culture Clostridium tetani from the wound would make the
diagnosis doubtful
B. Infection confers lifelong immunity
C. Clostridium-specific intravenous immunoglobulin is of no benefit once
spasms have been established.
D. Cephalic tetanus causes severe dysphagia

10. A school-going girl is listless on admission. On examination, she is lethargic


with punched out hyperpigmented lesions in places. Her temperature is 39.2°C,
and HR 150/min with cold peripheries. What is the most likely diagnosis?
A. Chicken pox
B. Measles
C. Meningococcaemia
D. Bacterial meningitis

11. A 15-year-old girl presented with a 12-hour history of fever and global headache.
On examination she was febrile (37.5oC). She was fully conscious. Mild neck
stiffness was noted but there were no other neurological signs. Cerebrospinal
fluid analysis showed:
Cell count 200 /mL (60% lymphocytes), Protein 0.8 g/L,
Glucose 4.3 mmol/L, Gram stain: No organisms seen
What is the most likely diagnosis?

A. Bacterial meningitis
B. Lymphomatous meningitis
C. Tuberculous meningitis
D. Viral meningo-encephalitis

12. Choose the best answer concerning management of HIV in children


A. All exposed under-5 children must be started on HAART regardless of HIV
status
B. Under five children must be started on the Tenofovir + Lamivudine +
Efavirenz
C. Infants must be started on Zidovudine + Lamivudine + Lopinavir/ritonavir
D. Children above five must be started on Tenofovir + Lamivudine + Efavirenz
MK

13. Which statement is true about treatment failure in HIV:


A. Clinical failure is the earliest
B. Virological failure is characterized by the failure of virus to replicate
C. Immunological failure is well expressed by the decline of CD4 count
D. Immunological failure is followed by virological

14. Concerning the diagnosis of HIV in children:


A. The rapid test is diagnostic at 1 year of age
B. In a child who never breast fed a negative DNA PCR is confirmatory negative
result
C. Rapid tests are never conclusive in under-5 children
D. A 2 years old child requires a DNA PCR to confirm the diagnosis

15. Which of these would be used to confirm diagnosis of Diabetes mellitus in a


symptomatic child?
A. Finding of 3+ ketonuria
B. An HbA1c of 7.8%
C. A fasting plasma glucose of 6.5mmol/L
D. An abnormal glucose tolerance test

16. Concerning childhood urinary tract infection:


A. It is commonly associated with vesico-ureteric reflux.
B. It is commonest in girls under one month of age.
C. It commonly presents with hematuria.
D. It is usually caused by Group B streptococcus.

17. Splenic enlargement is seen in all these, except:


A. Acute myeloid leukaemia
B. Typhoid
C. Idiopathic thrombocytopenic purpura
D. Infective endocarditis

18. Which of the following statements regarding kernicterus is incorrect?


A. Is due to unconjugated hyperbilirubinaemia
B. Presents with hypotonia in the early stages
C. May cause a chronic syndrome of athetosis and hearing loss
D. Is associated with contractures if not well managed
MK

19. A 7-year-old boy presents with crampy lower abdominal pain, loose stools and
anorexia of 3 days duration. The stools contain blood and slime and occur 20
times a day. On examination she appears lethargic, pale and has no jaundice. His
temperature is 37.6°, HR100/min, RR 15/min. He has a few purpuric spots on her
legs. What is the most likely diagnosis?
A. Marrow failure
B. Haemophilia
C. Haemolytic uraemic syndrome
D. Intussusception

20. Which of these is not a complication of Chicken pox?


A. Encephalitis
B. Cerebellar ataxia
C. Pneumonia
D. Myocarditis

21. A newborn baby develops difficult breathing 24 hours after birth associated with
persistent vomiting. He has no fever. On examination, his abdomen is scaphoid
and dullness on the left side of the chest. The most likely diagnosis is:
A. Necrotising enterocolitis
B. Hiatal hernia
C. Pyloric stenosis
D. Tracheo-oesophageal fistula with atresia

22. The following features about pyloric stenosis are false, except:
A. Females are more affected than males
B. Vomitus is at times made of fecal matters
C. Growth is rarely affected
D. The repair consists of myotomy

23. In a patient with acute rheumatic fever, the least likely of the following findings
is:
A. Arthritis
B. Bacteria in heart valves
C. Elevated ASO titer
D. Heart murmur
MK

24. The most frequent clinically significant residual lesion of acute rheumatic fever
is:
A. Aortic stenosis
B. Mitral disease
C. Myocardial hypertrophy
D. Pericardial adhesions

25. The following conditions are associated with microcephaly, except:


A. Vika virus
B. Congenital rubella
C. Post asphyxia encephalopathy
D. Congenital toxoplasmosis

26. Recognized findings in iron deficiency anemia include, except:


A. Microcytosis
B. Near normal RDW
C. Angular stomatitis
D. Megaloblasts

27. A cause of nutritional cardiomyopathy is:


A. Thiamine deficiency
B. Niacin deficiency
C. Vitamin C deficiency
D. Vitamin A deficiency

28. The normal hemostatic response to vascular damage depends on three major
factors, Except:**
A. The blood vessel wall
B. Stasis of blood flow
C. Circulating platelets
D. Blood coagulation

29. The following are specific to the intrinsic pathway, except:


A. Factor XII
B. Factor XI
C. Factor VII
D. Factor VIII
MK

30. Kerato-conjunctivitis is known to be as a complication of:


A. Measles
B. Rubella
C. Chicken Pox
D. Mumps

31. The following statements concerning management of neonatal jaundice are false,
except:
A. Phototherapy may result in diarrhea
B. Extra fluids are never required while the infant is receiving phototherapy
C. Breastfeeding is contraindicated in an infant receiving phototherapy
D. Bilirubin present in the urine on urinalysis indicates unconjugated jaundice

32. Pneumonia in childhood:


A. A viral infection is usually the forerunner.
B. Pneumocystis pneumonia often affects malnourished children.
C. Tuberculosis may present as lobar pneumonia
D. Streptococcal pneumonia is commonly an interstitial process.

33. Symptoms referable to the larynx include:


A. Inspiratory stridor
B. Aphonia
C. Wheezing
D. Expiratory stridor

34. The following statements about meningitis are false, except:


A. Cerebral blood flow is increased
B. There is inappropriate secretion of ADH
C. Serial head circumference measurement is important in infants
D. Non communicating hydrocephalus is frequent.

35. The following statements related to meningitis in children are true, except:
A. Nasopharyngeal bacterial carriage is often the source of infection.
B. Meningococcemia can manifest without meningeal signs.
C. Gram negative organisms are less common in infants.
D. A traumatic lumbar puncture is characterized by CSF ratio of 1
WBC/100RBCs.
MK

36. The following characteristics of malaria are true, except:


A. Plasmodium ovale is rare in Zambia
B. Diarrhoea and cough may be present
C. Relapses are common with P. falciparum
D. Nephrotic syndrome is associated with P. malariae

37. The following are features of complicated malaria, except:


A. Hypoglycaemia
B. Hypokalaemia
C. Respiratory distress
D. Abnormal bleeding

38. The following facts about diarrhoea in children are true, except:
A. Rotavirus is known to be responsible for at least 80% of diarrhea in infants.
B. It is persistent if it last more than 14 days and not caused by an infection.
C. Moderate dehydration is defined as loss of fluid of about 5 to 10%.
D. Selenium is essential as part of the management

39. Hemorrhagic disease of the newborn is known for the following, except:
A. May be prevented by prophylactic administration of Vitamin K.
B. Involves deficiency of factors II, VII, IX and X.
C. May be due to liver immaturity and lack of intestinal flora in the newborn.
D. Usually results in only mild skin bleeding.

40. The following conditions have wheezing in common, except:


A. Asthma
B. Bronchiolitis
C. Croup
D. Cystic fibrosis

41. The following statements are true about diabetes mellitus, except:
A. Honeymoon phenomenon can last a year
B. Fat is the last to be utilized in neoglucogenesis
C. Levels of potassium in the blood fall once insulin is commenced
D. Cerebral oedema is a common occurrence in children treated for DKA
MK

42. The following characteristics of diabetes mellitus are true, except:


A. Dextrose given to a known diabetic patient in coma with unknown blood sugar
levels may be life-saving.
B. Sodium bicarbonate is to be administrated as management of Diabetic
Ketoacidosis.
C. DKA is the most common first presentation in diabetes mellitus in children
D. Sugar is freely attached to Haemoglobin A1c.

43. A family has been using a live-in maid to care for their 3-year-old child while
away for work. The maid is diagnosed with pulmonary tuberculosis. Tuberculin
test of the child is negative. Which of the following is indicated for the 3-year-
old?
A. Six months of antituberculous treatment
B. Isoniazid chemoprophylaxis
C. Cotrimoxazole prophylaxis because the maid might be HIV positive
D. One month of streptomycin injections

44. A 4-year-old non immunized boy presents with bouts of coughing ending in
vomiting and a whoop. He has an absolute lymphocytosis. The probable cause of
his illness is
A. Diphtheria
B. Pertussis
C. Epiglottitis
D. Laryngitis

45. A 2-week-old baby girl is upset and has stridor. Her voice is hoarse and she has
a barking cough. She has a low-grade fever. There is history of rhinitis prior to
the cough. The most likely diagnosis for her condition is
A. Epiglottitis
B. Laryngo-tracheo bronchitis
C. Laryngo-tracheo-malacia
D. Papilloma virus
MK

46. A 3-year-old boy is very unwell. His temperature is 39°C and he is drooling
saliva as he is unable to swallow his secretions. He is toxic looking. There is a
soft stridor on closer examination. The most likely causative organism for his
condition is
A. Corynebacterium diphtheria
B. Respiratory syncytial virus
C. Parainfluenza virus
D. Haemophilus influenza

47. A 9-year-old girl presents with history of epistaxis for a period of one week. She
had been well prior to this except for a mild viral infection. Examination reveals
she also has petechiae and a few purpuric spots. She has no lymphadenopathy, or
hepatosplenomegaly. Reticulocyte count on investigation is 7%. What is the most
likely diagnosis?
A. Leukemia
B. Septicaemia
C. Immune thrombocytpaenia purpura
D. Hemophilia

48. A 5-year child presents to the emergency unit with history of sudden
deterioration in condition. It is not quite known what has happened to the child
as she was at the neighbors playing. On examination you notice that the child is
drowsy and almost choking on her own saliva. Examination of the pupils reveals
pin-point pupils. The most likely cause for this child’s condition is:
A. Salicylate poisoning
B. Lead poisoning
C. Organo-phosphate poisoning
D. Morphine poisoning
MK

49. During labour, the fetal heart tracing begins to show late decelerations. To
expedite the delivery, the obstetrician ruptures the membranes. The infant is
delivered 1 hour later and appeared cyanotic and limp requiring extensive
resuscitation. The initial physical examination reveals an infant who has poor
tone and deep tendon reflexes. At 14 hours of life, the infant develops seizures.
The perinatal course described above is most consistent with the following;
A. Development of subarachnoid hemorrhage
B. Birth asphyxia with hypoxic ischaemic encephalopathy
C. Respiratory distress syndrome
D. Intraventricular hemorrhage

50. The neonate described above in Q 49 is likely to have the following complication
A. Spastic Quadriplegic cerebral palsy
B. Diplegic cerebral palsy
C. Choreo-athetoid cerebral palsy
D. Ataxic cerebral palsy

51. A neonate presents with hepatosplenomegaly, the following are tests required
for management, except:
A. RPR
B. RDT for malaria
C. Rapid HIV test
D. Full blood count

52. The following statements about HIV in infancy are true, except:
A. Nucleic Acid Testing (NAT) is mandatory at birth and 6 months of age
B. Asymptomatic exposed infants should not be commenced on Antiretroviral
prophylaxis
C. All exposed infants are to undergo the screening rapid test at first contact
D. Exposed NAT negative infants need to be maintained on ART prophylaxis as
long as they are still breastfeeding

53. The following conditions are stage III, except:


A. Extensive chicken pox
B. Oral candidiasis
C. Lymph node tuberculosis
D. Oral hairy leucoplakia
MK

54. The following are suggestive facts about HIV infection in a newborn, except:
A. Prematurity
B. Intra-uterine growth restriction
C. Hepatomegaly
D. Parotid enlargement

55. The following are possible features of a rapidly progressive HIV disease in an
infant, except:
A. Generalised lymphadenopathy
B. Oral thrush during the neonatal period
C. Hepatosplenomegaly
D. Low birthweight

56. Choose the correct line of management for antiretroviral prophylaxis for exposed
infants:
A. Stavudine, Lamuvidine and Efavirenz
B. Zidovudine, Lamuvidine and Nevirapine
C. Abacavir, Emtricitabine and Nevirapine
D. Zidovudine, Lamuvidine and Efavirenz

57. The following are known risk factors for tuberculosis in children, except:
A. Household contact with a newly diagnosed smear-positive case
B. Age less than 5 years
C. History of tuberculosis in the extended family
D. Severe malnutrition.

58. The following are true about management of malnutrition in children, except:
A. Shock should absolutely be managed with intravenous fluids
B. Zinc is not needed in diarrhoea because it is part of F75 feeds
C. Whole blood is better than packed cells in oedematous malnutrition
D. Compared to ORS, ReSoMal contains more potassium and less sodium
MK

59. The most logical sequence of events of Vitamin A deficiency is as follow:


A. Corneal scarring, night blindness then staphyloma
B. Nictalopia, xerosis then corneal ulceration
C. Corneal ulceration, conjunctival injection then total blindness
D. Bitot spots ,nictalopia, then corneal scarring

60. The following are acceptable definitions of failure to thrive, except:


A. A child under 6 months who has not grown for 2 months
B. A child over 6 months has not grown for 3 months
C. A weight- for- age that falls below the 5th percentile on multiple occasions
D. A weight deceleration that crosses one major percentile line on a growth chart

61. The following conditions may lead to failure to thrive, except:


A. Coeliac disease
B. Cystic fibrosis
C. Long gut syndrome
D. Hyperthyroidism

62. The most important parameter to help detect failure to thrive is:
A. Diet
B. Growth curve
C. Developmental milestones
D. Bowel habit and type of stool

63. Human milk contains more of the following than cows' milk formula:
A. Lactalbumin
B. Calcium
C. Iron
D. Folic acid

64. The following are benefits of breastfeeding, except:


A. Provides superior nutrition for optimum growth.
B. Protects against malaria
C. Provides adequate water for hydration
D. Protects against infection and allergies
MK

65. The following anti-infective substances are present in mature breast milk,
except:
A. IgG
B. Bifidus factor
C. Transferrin
D. Lymphocytes

66. You are asked to counsel a woman who is planning to have another baby after
her first baby was born with spina bifida. Which one of the following
preconception management options is most likely to reduce the risk in any
subsequent pregnancy?
A. Sodium valproate
B. Thiamine
C. Folic acid
D. Vitamin B12

67. The following statements concerning cephalhaematoma are true, except:


A. Does not cross sutures
B. Fluctuation is not present
C. Present at birth
D. Presents few days after birth

68. The characteristics of caput succedaneum include all, except:

A. Crosses midline
B. Crosses suture line
C. Takes weeks to resolve
D. It is diffuse oedematous swelling of soft tissue of the scalp

69. Which one of the following is false concerning malnutrition:


A. F75 milk is introduced at admission
B. They present with hypokalaemia and hypernatraemia
C. Dermatosis is an indication for admission
D. Patients may present with non-pitting pedal oedema
MK

70. The parameter that best indicates chronicity of malnutrition is:


A. Head circumference
B. Height
C. Weight
D. All the above

71. Refeeding syndrome may present with the following, except:


A. Respiratory distress
B. Fever
C. Convulsions
D. Oliguria
72. The following are characteristics of Sickle Cell Disease, except:
A. Predisposes to pneumococcal septicaemia and salmonella osteomyelitis
B. Frequently results in a palpably enlarged spleen
C. Aplastic crisis is often due to bone marrow ischemia
D. Dactylitis occurs early in life

73. The following findings are common in Sickle Cell Disease, except:
A. Jaundice is not necessarily pronounced in sequestration crisis
B. Clubbing is not a common finding
C. Acute chest syndrome is often associated with sepsis
D. Asplenia causes susceptibility to both viral and bacterial infections

74. The following is true of Sickle Cell Disease, except:


A. Affected subjects are prone to malaria
B. Bossing is due to avascular necrosis of the bones
C. Hyposthenuria is a result of infarction of renal medulla
D. Affected subjects may experience delayed puberty

75. The following are true about hematologic changes in Sickle Cell Disease, except:
A. Anisocytosis is a very rare blood finding.
B. Target cells are commonly found on the peripheral blood smear
C. Iron deficiency may occur despite re-use after hemolysis
D. Red blood cell distribution width (RDW) is usually increased
MK

76. The following are complications arising from sickle cell disease, except:
A. Cholecystitis
B. Phimosis
C. Leg ulcers
D. Transient ischaemic attack

77. The normal hemostatic response to vascular damage depends on three major
factors, except:
A. The blood vessel wall
B. Stasis of blood flow
C. Circulating platelets
D. Blood coagulation

78. The following are symptoms and signs of congestive cardiac failure, except:
A. Poor feeding
B. Sweating
C. Splenomegaly
D. Central cyanosis

79. The following clotting factors are specific to the intrinsic pathway, except:
A. Factor XII
B. Factor XI
C. Factor VII
D. Factor VIII

80. The following are common manifestations of carditis in rheumatic heart


disease, except:
A. Cardiomegaly
B. Cardiac failure
C. Valvulopathy
D. Pericardial effusion

81. All of the following are included in the revised Jones Major criteria, except:
A. Maculopapular rash
B. New murmur (carditis)
C. Migrating polyarthritis
D. Chorea
MK

82. A 7-year-old girl presents with a tender and swollen right knee as well as a more
recently appearing swollen left wrist. She also has a fever. This patient fulfills
which of the following modified Jones criteria?
A. 1 Major 1 minor
B. 1 Major 2 minors
C. 2 Majors
D. 2 Minors

83. Rheumatic heart disease is often associated with the following, except:
A. Infective endocarditis
B. Recent infection with streptococcus
C. Huntington chorea
D. Pericarditis

84. The following cardiac defects are characteristic of tetralogy of Fallot, except:
A. Pulmonary stenosis
B. Transposition of great arteries
C. VSD
D. Right ventricular hypertrophy

85. Concerning fetal circulation, blood flows from the placenta through which route?
A. Umbilical arteries, ductus venosus, inferior vena cava and right ventricle
B. Umbilical vein, ductus venosus, inferior vena cava and right atrium
C. Umbilical veins, ductus arteriosus, superior vena cava and foramen ovale
D. Umbilical arteries, ductus arteriosus, portal vein and ductus arteriosus
86. A patient with newly diagnosed acute myeloid leukemia is initially seen with
neutropenia and a temperature of 40oC should be managed with which of the
following measures?
A. Prompt institution of chemotherapy because the fever is most likely due to
leukemia
B. Prompt procurement of cultures and initiation of broad-spectrum parenteral
antibiotics
C. Administration of granulocyte transfusions to correct the neutropenia
D. Extensive search for an underlying infection and withholding antibiotics until
one is found
MK

87. A 3-year-old child presents with history of an abdominal swelling for 2 months
duration. On examination, the patient is cachexic, pale and had bleeding around
the eyes. She also has multiple cervical lymphadenopathy and lumps on the head.
You also note that the child is bleeding under the mucous membrane of her oral
cavity. Her blood pressure is 170/100mmHg. What is the most likely diagnosis?
A. Wilms tumour
B. Leukemia
C. Neuroblastoma
D. Non-Hodgkin’s lymphoma

88. The following are common findings in idiopathic nephrotic syndrome, except:
A. Abdominal pain
B. Anaemia
C. Oedema
D. Hypertension

89. In the nephrotic state, serum lipid levels are elevated for the following reason(s):
A. Hypoalbuminemia stimulates hepatic protein synthesis, including
lipoproteins.
B. Lipid catabolism is increased as a result of reduced plasma levels of
lipoprotein lipase, related to the increased urinary loss of this enzyme.
C. A and B are true
D. None of the above

90. The following conditions are possible causes of nephrotic syndrome, except:
A. Systemic lupus erythematous
B. Syphilis
C. Malaria
D. Tuberculosis

91. The following statements are true about post-streptococcal glomerulonephritis,


except:
A. It is characterized by raised C3 in the acute phase
B. It arises mainly from infected skin sores
C. Hypertensive encephalopathy is a recognized complication
D. Treatment with penicillin is mainly prophylactic
MK

92. Management of acute glomerulonephritis include the following, except:


A. Fluid restriction
B. Salt restriction
C. A high protein diet
D. Antihypertensive drugs

93. The following are characteristic features of acute glomerulonephritis, except:


A. Oedema
B. Uraemia
C. Heavy Proteinuria
D. Hypertension

94. These haematological changes are characteristic of neonatal sepsis, except:


A. Thrombocytopaenia
B. Persistently high White Blood Cell count
C. Anaemia
D. Elevated C Reactive Protein is diagnostic

95. The following are risk factors for Early Onset Neonatal Sepsis, except:
A. Birth asphyxia
B. Low birth weight
C. Congenital heart disease
D. Maternal fever

96. Late Onset Neonatal Sepsis is characterized by the following, except:


A. Group B Streptococcus is no longer among the causative organisms.
B. This infection is more localized
C. E.coli is the predominant causative organism
D. Occurs two weeks after birth

97. The following statements are true about Early Onset Neonatal Sepsis, except:
A. Occurs within the first 7 days of life for the full term neonate
B. Group B Streptococcus is the predominant causative organism
C. Has lower mortality compared to late onset sepsis
D. Involves more than one body systems
MK

98. The following are true about respiratory distress syndrome, except:

A. Bronchopulmonary dysplasia is partly as a result of high-pressure oxygen


administration.
B. Surfactant is made of sphingomyelin and lecithin
C. Infants of diabetic mothers are prone irrespective of gestational age.
D. Hydrocortisone given during labour is preventive

99. Only one statement is true about respiratory distress syndrome:

A. Affects only infants born under 34 weeks


B. Typically causes lobar collapse on chest X-ray
C. Maternal steroid therapy improves outcome
D. Is prevented by surfactant therapy

100. Common indications that a neonate has suffered intrapartum hypoxia are as
follow, except:
A. Hypotonia
B. Polyuria
C. Seizures
D. Excessive crying

101. Which one of the following is the most important risk factor in the etiology of
intraventricular haemorrhage (IVH)?
A. Coagulation disorder
B. Intermittent positive-pressure ventilation
C. Pneumothorax
D. Extreme prematurity

102. Children with cerebral palsy have these in common, except:


A. Are usually born prematurely
B. Have physical signs that change with time
C. Often fail to thrive
D. Kernicterus is often associated with deafness
MK

103. Cerebral palsy can be best defined as:


A. Stiffness of all four limbs in an infant as a result of an insult on the fetal or
infant brain
B. Limitation in the range of limbs movement caused by an infectious process
on the fetal or infant brain.
C. A condition characterised mainly by mental retardation as a result of an insult
on a developing fetal or infant brain
D. Permanent disorder of movement and posture due to non –progressive insult
on a developing infant brain

104. A 2-month-old infant, with a tinge of jaundice, is being treated for sepsis. He
is reported to have fitted twice. A lumbar puncture was performed to rule out
meningitis. He is later on diagnosed with cerebral palsy. What is the most likely
type?
A. Athetoid type
B. Ataxic type
C. Spastic type
D. Choreo-athetoid type

105. In gross prematurity, cerebral palsy is often as a result of:


A. Kernicterus
B. Birth asphyxia
C. Periventricular leukomalacia
D. Respiratory distress syndrome

106. The following statements about cerebral palsy are true, except for:
A. Can be detected antenatally
B. The cause is often unknown
C. There is no known cure
D. Genetic influences may play a role
MK

107. The following conditions cause unconjugated hyperbilirubinaemia, except:


A. Congenital spherocytosis
B. Choledocal cyst
C. Kernicterus
D. Neonatal sepsis

108. Phototherapy is indicated in the following conditions, except:


A. Congenital spherocytosis
B. Neonatal hepatitis
C. Rhesus incompatibility
D. Cholelithiasis

109. The following conditions increase the risk of developing kernicterus in a


neonate with jaundice, except:
A. Cephalhaematoma
B. Dehydration
C. Prematurity
D. Birth asphyxia

110. In a child whose jaundice begins at less than 24 hours of age, the following
diagnoses should be considered, except:
A. CMV infection
B. ABO incompatibility
C. Hereditary spherocytosis
D. Hypothyroidism

111. Which of the following is a clinical feature of congenital rubella:


A. Conductive deafness
B. Cataract
C. Hydrocephalus
D. Dental defects

112. Choose the only wrongly matched congenital infection to its complication in
the newborn:

A. Rubella Patent ductus arteriosus


B. Cytomegalovirus Hydrocephalus
C. Syphilis Snuffles
D. Herpes simplex Encephalitis
MK

113. A 1-week-old neonate presents with bloody nasal discharge, hepatomegaly and
hemolytic anaemia. Which of the following is the most likely additional feature
in this infant to complete the diagnosis of a well-known congenital infection?

A. Hydrocephalus
B. Periostitis
C. Seizures
D. Cataracts

114. Of the following, the congenital infection most associated with hydrops fetalis
is:
A. Rubella
B. HIV
C. Syphilis
D. Toxoplasmosis

115. The following true of Measles, except:


A. The rash occurs as the fever subsides
B. Koplik’s spots are pathognomonic
C. Antibodies from the mother reduce the likelihood of measles infection
before 6 months of age
D. Incubation period is 21 days

116. Complications of Measles include the following, except:


A. Myocarditis
B. Laryngo-tracheo-bronchitis
C. Acute encephalitis
D. Otitis media

117. If not well managed, severe measles infection may lead to the following
condition, except:
A. Fulminant hepatitis
B. Cancrum oris
C. Tuberculosis
D. Severe malnutrition
MK

118. A 20-week-old infant missed his routine immunisations. He is brought to the


clinic today. He is to be given the following vaccines, except:
A. Diphtheria
B. Rotavirus
C. Hepatitis
D. PCV

119. Regarding BCG vaccine, the following are true, except:


A. Provides more consistent protection against TB meningitis than pulmonary
TB
B. Typically given in the left upper arm
C. It is given as intradermal
D. Contra-indicated in symptomatic HIV infection

120. Membranous pharyngitis, enlarged anterior cervical lymph nodes and ‘bull
neck’ appearance are characteristics of the condition below :
A. Measles
B. Tuberculosis
C. Diphtheria
D. Pertussis

121. The following DO NOT apply to Rotavirus gastroenteritis, except:


A. It typically affects infants older than 18 months
B. Blood and mucus is found in the stools of about 50% of affected babies
C. It occurs most often in the hot months of the year
D. Protection is conferred by breast feeding

122. The following are true of Down’s syndrome, except:


A. Male subjects may be infertile
B. 4% of cases are as a result of translocation
C. Subjects with mosaicism seem to have a better IQ
D. Umbilical hernia is common

123. Subjects with Klinefelter syndrome have this in common, except:


A. Males may be infertile
B. Hypergonadism is the hallmark due to XXY
C. They display some learning impairment
D. They are characterised by tall stature
MK

124. Turner’s syndrome is characterised by the following, except:


A. Caused by the loss of genetic material from one of the sex chromosomes
B. Affected subjects are prone to diabetes
C. In a very small proportion, male subjects may be affected
D. Estrogen replacement therapy is indicated

125. The following are true about hydrocephalus, except:


A. Toxoplasmosis is a known cause
B. Nephritis is a possible complication when shunted
C. Cracked pot is a characteristic sign on examination
D. Sun rising eyes is obvious when intracranial pressure is raised

126. The following statements are true about meningitis, except:

A. Group B Streptococcus more common in infants


B. Fluid restriction at all times due to SIADH
C. Steroids indicated to prevent hearing loss when due to H. influenzae
D. Shock is a likely presentation

127. Lumbar puncture is contra indicated in the following circumstances, except:


A. Frequent fits
B. Bulging anterior fontanel
C. Low pulse rate associated with high blood pressure
D. Anisocoria

128. Common causative organisms in children are:


A. Hemophilus influenzae b, Streptococcus pneumoniae and Klebsiella.
B. Hemophilus influenza b, Listeria monocytogenes and Streptococcus
pneumoniae.
C. Staphylococcus epidermidis, Staphylococcus aureus and Streptococcus
pneumoniae.
D. Hemophilus influenzae b, Streptococcus pneumoniae and Neisseria
meningitides.
MK

129. The following are true of Malaria, except:


A. Plasmodium falciparum invades both mature and young red blood cells
B. Plasmodium vivax is also found in Zambia
C. Children with sickle cell anaemia are immune to malaria
D. Artesunate is the first line for treatment of severe malaria in Zambia

130. The following facts are true of malaria, except for:


A. One generalized tonic clonic is characteristic of severe malaria
B. Plasmodium malariae causes nephrosis
C. Meningitis should be excluded in case of severe malaria
D. Young children and pregnant women are more vulnerable

131. The following statements are true concerning diabetes mellitus type I, except
A. The destruction of α cells in the pancreas leads to absolute insulin deficiency.
B. Glycosylated haemoglobin helps in the monitoring of management.
C. Diabetic keto-acidosis may be the first presentation in children
D. Enuresis should be alluded to when investigating diabetes mellitus

132. The following signs and symptoms of diabetes mellitus Type I are very
common in children, except:
A. Bed-wetting
B. Excessive hunger
C. Loss of weight
D. Excessive thirst

133. Management of Diabetic Ketoacidosis consists of the following components,


except:
A. Correction of shock
B. Potassium replacement
C. Urgent bicarbonate replacement
D. Treatment of infection
MK

134. The following statements regarding Somogyi effect as a result of diabetes in


children are correct, except:
A. It may occur even in the absence of insulin therapy
B. Blood sugar levels are high in the morning before breakfast
C. Cortisol and catecholamines are known to counteract this phenomenon
D. It occurs mainly in poor controlled diabetes

135. The following are true of congenital megacolon, except:


A. Abdominal distension in the newborn period may be the presenting feature
B. Diagnosis can be by rectal examination.
C. The affected segment is grossly dilated on barium enema
D. It is commonly associated with Down’s syndrome

136. A 4-week-old baby presents with jaundice. The following statements are false,
except:
A. No investigations are necessary if the baby is entirely breastfed
B. It is an indication to stop breast feeds
C. Conjugated hyperbilirubinaemia is never physiological
D. Breast milk jaundice is associated with pale stools

137. Only one statement is true about congenital pyloric stenosis:


A. Presents in the first few weeks after birth
B. Can cause hyperchloraemic alkalosis
C. Is inherited as an autosomal recessive
D. Typically presents with bile stained vomiting after feeds

138. Intussusception is characterised by the following, except:


A. Is more common in the first 3 months of life
B. May present with red currant jelly stools in the first few hours
C. In well patients, can usually be reduced by Barium enema
D. Should always be investigated fully for the cause after reduction
MK

139. Congenital diaphragmatic hernia is characterised by the following, except:


A. Usually occur through the foramen of Bochdalek
B. Are more common on the right than the left
C. Are often associated with other congenital anomalies
D. Frequently present with respiratory distress.

140. These are characteristics of febrile convulsions, except for:


A. They are benign and generally not associated with neurological sequelae
B. The mainstay of investigation and treatment is to rule out infection/
infestation
C. Brain CT scan is a compulsory indication to rule out space occupying lesion.
D. Children below 6 months are never affected

141. A 15 months old child, is admitted with severe dehydration as a result of


watery diarrhoea and weighs 13 kg. She has been successfully rehydrated and
is out of shock. Her 24-hour maintenance fluid should be:
A. 1300 mls
B. 1400 mls
C. 1150 mls
D. 1050 mls

142. She is then given adjuvant treatment for the diarrhoea. Which one of the
following is she not supposed to be given:
A. Plain water
B. Fresh milk
C. Zinc sulphate
D. Zinc oxide

143. The best oral rehydration fluid for this toddler would be:
A. ReSoMal
B. Low osmolarity ORS
C. High osmolarity ORS
D. Homemade ORS
MK

144. The following are true about birth asphyxia, except:


A. The cardiorespiratory systems often provide the first signs of end-organ
dysfunction.
B. Myocardial ischaemia and poor contractility are as a result of acidosis
C. Disseminated intravascular coagulation is always due to severe sepsis
D. Thrombocytopaenia is the first sign of marrow suppression

145. The following drugs are used to manage severe birth asphyxia, except:
A. Dexamethasone
B. Phenobarbitone
C. 10% Dextrose
D. Mannitol

146. The following are known characteristics of Broncho-pulmonary dysplasia,


except:
A. Persistent oxygen dependency for up to 28 days
B. Administration of oxygen via CPAP may worsen the condition
C. May be caused by meconium aspiration
D. Barotrauma is a known contributing factor

147. Following babies are at risk of developing necrotizing enterocolitis, except:


A. Premature infants
B. Infants who are fed concentrated formulas
C. Infant of diabetic mother
D. Infants who have received blood exchange transfusions

148. Characteristic features of physiological jaundice include all of the following


except:
A. Onset after the first 24 hours of life
B. Disappearance by the tenth day of life
C. A serum bilirubin level below 250 mmol/L
D. Associated anaemia due increase breakdown of extra RBCs
MK

149. Delayed speech in a 5-year-old with normal motor and adaptive development
is most likely due to:
A. Mental retardation
B. Deafness
C. Cerebral palsy
D. Kernicterus

150. The following is true about child’s growth, except for:


A. Average weight by 8 months of age is 8 kg
B. Average height by 3 years of age is 95 cm
C. Average fronto-occipital circumference by 3 months of age is 40 cm
D. Average daily weight gain for a 2-week neonate is 75 g

151. Signs constituting severe malaria include all of the following, except:
A. Convulsions
B. Pulmonary hypertension
C. Haemoglobinuria
D. Jaundice

152. Causes of stridor in children include the following, except:


A. Epiglottitis
B. Retropharyngeal abscess**
C. Bronchiolitis**
D. Foreign body

153. The following findings are common in Sickle Cell Disease, except:
A. Jaundice is not a common feature in sequestration crisis
B. Asplenia causes susceptibility to both viral and bacterial infections
C. Jaws are often affected by extra-medullary erythropoiesis
D. Acute chest syndrome is often associated with sepsis

154. The following are true about haematologic changes in Sickle Cell Disease,
except:
A. Poikylocytosis is characterized by spiked red blood cells
B. Target cells are commonly found on the peripheral blood smear
C. Iron deficiency may occur despite re-use after haemolysis
D. Red blood cell distribution width (RDW) is usually increased
MK

155. The following is true of Measles, except:


A. Fever reaches its zenith as the rash erupts
B. Immunity is life-long if acquired from a mother who had a severe infection in
her childhood.
C. Infection before 6 months of age is possible
D. Incubation period is about 14 days

156. A healthy 9-month old male infant presents to the emergency room with low
grade fever, difficult breathing for one night, and breastfeeds intermittently. On
examination, he is wheezing, has intercostal and sub-costal recessions and
saturating at 91% at room air. What is the most likely diagnosis?
A. Epiglottitis
B. Severe bacterial pneumonia
C. Bronchiolitis
D. Aspiration pneumonia

157. Which of the following complications of diabetic ketoacidosis has the highest
mortality rate?
A. Cerebral oedema
B. Hypokalaemia
C. Hyperglycaemia
D. Acidosis

158. A newborn is brought to the delivery room. After assessing the infant’s heart
rate, color, and respiratory effort, a decision is made that the infant should receive
ventilatory assistance. After 30 seconds of this therapy, CPAP is initiated. The
most likely APGAR score at this point is
A. 9
B. 8
C. 5
D. 2
MK

159. A mother is at 40 weeks gestation. The dating by time of last menstrual cycle
agrees with an early ultrasound. At birth, the infant weighs 1.5kg and has a head
circumference that is abnormally small. The platelet count is also low. What is
the most likely cause of the poor growth of this child?
A. Placental insufficiency
B. Intrauterine infection in the first trimester
C. Neonatal Disseminated Intravascular coagulopathy
D. Neonatal coagulopathy

160. An infant is born prematurely at 28 weeks gestation and weighs 1.28kg. The
infant is started on enteric feeds at 2 days of age. Three days later, he is not
tolerating his feeds. The signs and symptoms exhibited by the infant that make
you suspect he has neonatal necrotizing enterocolitis include the following,
except
A. Apnoea
B. Diarrhoea
C. Pneumothorax
D. Jaundice

161. An infant born 30 weeks gestation begins to experience apnea on the second
day of life. All of the following should be included in the initial management of
this infant, except:
A. Evaluation for evidence of infection or intracranial hemorrhage
B. Check that the environmental temperature is above 25 Celsius
C. Complete blood count, arterial blood gas studies, and plasma glucose and
electrolyte management
D. Assessment for evidence of hydrocephalus

162. The following are known facts about cerebral palsy, except:
A. May affect the entire body, yet have no impact on intelligence
B. May cause precocious puberty
C. Spastic type has highest likelihood of mental retardation
D. Gastroesophageal Reflux Disease
MK

163. A 2-year-old girl, well 24 hours ago, is brought to the emergency room in
shock. She has no history of trauma. On further examination, she is extremely
pale with an extreme large spleen. Her thick smear reveals no malaria and she is
afebrile. The most plausible cause of is the most likely explanation of her
condition is
A. Spontaneous massive internal haemorrhage
B. Cardiogenic shock
C. Splenic sequestration crisis in sickle cell anemia
D. Hypersplenism

164. These are features commonly found in newborn term infants with
uncomplicated down’s syndrome except:
A. Inguinal hernia
B. Nystagmus
C. Brachycephaly
D. Hypothyroidism

165. Precious, a 13-year-old girl a resident of Kamwala south presents with fever
for 2 weeks and weight loss. What would be your next appropriate step?
A. Start anti-tuberculous treatment immediately
B. Start the adolescent on F75
C. Get a more detailed history
D. Start the patient on Ciprofloxacin as this could be enteric fever

166. Precious, a 8 year-old-girl presents with body hotness for 3 weeks, poor
appetite, weight loss and sweating. Her differential diagnosis includes the
following except
A. Disseminated TB
B. Typhoid fever
C. Acute leukemia
D. Severe malaria
MK

167. The following are well-known causes of neonatal jaundice except:


A. Hemolytic disease of the new born
B. Hemorrhagic disease of the new born
C. TORCH infections
D. Biliary atresia

168. Which of the following statements concerning immunization is true?


A. The presence of a coryzal illness and fever of 37.9oC is a contraindication
B. Rotavirus is respectively given at 6, 10 and 14 weeks of age
C. Pneumococcal vaccine is the last vaccine to be introduced on the Zambian
EPI
D. A history of severe eczema is a contraindication

169. Which of the following statements regarding examination of the respiratory


system is correct?
A. Bronchial breath sounds heard in the 5th right intercostal in the subscapular
region indicates a right middle lobe pneumonia
B. A persistent wheeze with reduced breath sounds is diagnostic of asthma
C. Pectus excavatum is mostly indicative of underlying lung pathology
D. The ribs of children are more horizontal than that of adults

170. The following are known causes of polyhydramnios except:


A. Maternal diabetes
B. Trisomy 21
C. Hydrops fetalis
D. Renal agenesis

171. The following are known causes of oligohydramnios except:


A. Posterior urethral valves
B. Polycystic kidneys
C. Spina bifida
D. Premature rupture of membranes
MK

172. Mr and Mrs Woods are planning another pregnancy. Mr Wood’s mother and
Mrs Woods’ father are siblings. They have a healthy son, but had 2 children who
died in early infancy. They were told that it was due to “metabolic condition”.
This condition is likely due to
A. Multifactorial inheritance
B. Autosomal recessive inheritance
C. X-linked inheritance
D. Chromosomal translocation

173. Phiri is the youngest of a family of 5 siblings. Both parents are in their early
thirties. The mother is very worried about Albert’s long hours of sleep, his “being
too weak” and his tongue that is constantly out, and a big umbilicus. Phiri’s
condition is mostly due to
A. Multifactorial inheritance
B. Autosomal recessive inheritance
C. Chromosomal translocation
D. Chromosomal non-disjunction

174. Little Nora, newly born, has extra digits on both hands. The mother says she
had the same. This polydactyly is as a result of:
A. X-linked inheritance
B. Autosomal recessive inheritance
C. Autosomal dominant inheritance
D. Chromosomal non-disjunction

175. Rufai comes home with a swollen knee. He is then rushed to the nearest clinic
where the doctor performs a tap that yields frank blood. He is the only boy among
3 girls. Mother says her only brother died in his tender age after bleeding
uncontrollably as a result of circumcision performed by a traditional healer in the
village. Rufai’s condition is likely due to
A. X-linked inheritance
B. Autosomal recessive inheritance
C. Autosomal dominant inheritance
D. Chromosomal non-disjunction
MK

176. Manjimela has obvious big foot and the father amusingly states that God gifted
the family with this unique feature which makes them stand out. This is likely as
a result
A. X-linked inheritance
B. Polygenic phenomenon
C. Autosomal inheritance
D. Chromosomal deletion

177. Immediate complications of birth asphyxia include the following


A. Acute tubular necrosis
B. Surfactant deficiency
C. Necrotizing enterocolitis
D. Hemothorax

178. The following statements are true except:


A. Walking after the age of 20 months should be considered as a delay
B. The average Zambian child weighs 9kg at 12 months of age
C. Hearing is the highly developed sense in neonates
D. Rapid growth and development are observed in the first year of life

179. Which of the following normal infant values is incorrect?


A. Average weight at 6 months: 6kg
B. Average height at 2 years: 94cm
C. Average head circumference at 1 year: 45cm
D. Average heart rate in a neonate: 140/min

180. The following are as a result of re-feeding syndrome in severe malnutrition


except
A. Constipation
B. Tachycardia
C. Seizures
D. Renal failure
MK

181. These are characteristics of febrile convulsions except


A. They are almost benign and generally not associated with neurological
sequelae
B. The mainstay of investigation and treatment is to rule out infection
C. Brain CT scan is a compulsory indication to rule out space occupying lesion
D. They usually last up to 15 minutes

182. A newborn is brough to the delivery room. After assessing the infants heart
rate, color and respiratory effort, a decision is made that the infant should receive
ventilatory assistance. After 30 seconds of this therapy, cardiac compressions are
initiated. The most likely apgar score at this point is
A. 9
B. 7
C. 6
D. 2

183. An infant born 30 weeks begins to experience apnea on the second day of life.
All of the following should be included in the initial management of this infant
except:
A. Evaluation for evidence of infection or intracranial hemorrhage
B. Bronchoscopy to evaluate the upper and lower airway
C. Complete blood count, arterial blood gas studies and plasma glucose and
electrolyte management
D. Check that the environmental temperature is above 25 celsius

184. Signs constituting severe malaria include all the following except
A. Convulsions
B. Polycythemia
C. Hemoglobinuria
D. Jaundice

185. The following are known complications of cerebral palsy


A. Convulsions
B. Contractures
C. Gastroesophageal reflux disease (GERD)
D. Precocious puberty
MK

186. These are features commonly found in newborn term infants with
uncomplicated Down’s syndrome except
A. Nystagmus
B. Brachycephaly
C. Down slanting palpebral fissure
D. Hypothyroidism

187. The following statements related to neonatal sepsis are true, except:
A. Thrombocytopenia frequently associated with sepsis
B. WBC may be high, low or normal
C. Timing of sampling has very little bearing of the results
D. Persistent low WBC more predictive of sepsis than elevated WBC

188. Critical to the management of early onset neonatal sepsis are the following,
except:
A. Dopamine
B. Normal saline
C. Fresh Frozen Plasma
D. Hydrocortisone

189. The following infections are common in both HIV infected and uninfected,
except:
A. Otitis media
B. Bronchectasis
C. Tuberculosis
D. Diarrhoea

190. Antiretroviral prophylaxis in neonate born an ART naïve mother is as follow:


A. ABC + 3TC + NVP
B. ABC + 3TC + LPV/r
C. AZT + 3TC + NVP
D. AZT + 3TC + EFV
MK

191. A female adolescent, weighing 40 kg, is to be commenced on ART. All the


following drugs can be used in combination, except:
A. Dolutegravir
B. Emtricitabine
C. Tenofovir
D. Efavirenz
192. Steroids are indicated in the following forms of tuberculosis in children
except:
A. Pericardial effusion
B. Tuberculosis meningitis
C. Pleural effusion
D. Airway obstruction by Tuberculous adenitis
193. The following conditions are highly suggestive of tuberculosis, except:
A. Painful cervical lymph node
B. Cold abscess
C. Gibbus
D. Acute onset of raised intracranial pressure
194. The most important parameter to help detect failure to thrive is:
A. Diet
B. Growth curve
C. Developmental milestones
D. Bowel habit and type of stool

195. The following are benefits of breastfeeding, except:


A. Provides superior nutrition for optimum growth.
B. Provides adequate water for hydration
C. Protects against malaria
D. Protects against infection and allergies

196. The following nutrients are particularly critical for brain development, except:
A. PUFA
B. Iron
C. Copper
D. Sodium
MK

197. The following statements concerning cephalhaematoma are true, except:


A. Birth asphyxia might be the cause
B. May result into pathological neonatal jaundice
C. Leaves no cranial deformity once resolved
D. Presents few days after birth

198. Which one of the following is true concerning malnutrition:


A. Both F100 and F75 feeds can be introduced simultaneously at admission
B. They present with hypokalaemia and hypernatraemia
C. Dermatosis is an indication for admission
D. Patients may present with non-pitting pedal oedema

199. Antibiotics in the management of Acute Chest Syndrome include the


following, except:
A. Cephalosporines
B. Penicillins
C. Macrolides
D. Chloramphenicol

200. The following hematological findings are common in Sickle Cell Disease,
except:
A. Low Haematocrit
B. Normal MCV
C. Low C3 Complement
D. Low platelet count

201. The following findings are common in Sickle Cell Disease, except:
A. Sequestration crisis is always associated circulatory collapse
B. Clubbing is not a common finding
C. Acute chest syndrome is often associated with sepsis
D. Asplenia causes susceptibility bacterial infections

202. The following are symptoms and signs of congestive cardiac failure, except:
A. Poor feeding
B. Sweating
C. Splenomegaly
D. Central cyanosis
MK

203. The following are common manifestations of carditis in rheumatic heart


disease, except:
A. Cardiomegaly
B. Cardiac failure
C. Valvulopathy
D. Pericardial effusion

204. All of the following are included in the revised Jones Major criteria, except:
A. Maculopapular rash
B. New murmur (carditis)
C. Migrating polyarthritis
D. Chorea

205. The following are complications of Tetralogy of Fallot, except:


A. Anemia
B. Brain abscess
C. Thrombocytopenia
D. Jaundice

206. Management of Tetralogy of Fallot include the following, except:


A. Exchange blood transfusion
B. Antibiotics
C. Morphine
D. Erythropheresis

207. The following are common findings in idiopathic nephrotic syndrome, except:
A. Abdominal pain
B. Anaemia
C. Oedema
D. Hypertension

208. The following are true of Down’s syndrome, except:


A. Nearly of male subjects are infertile
B. 95% of cases are as a result of translocation
C. Subjects with mosaicism seem to have a better IQ
D. Umbilical hernia is common
MK

209. The commonest mechanism of genetic abnormality in a case of Trisomy 21 is:


A. Non disjunction in paternal gamete
B. Translocation to chromosome 14
C. Translocation to any autosome
D. Non disjunction in maternal gamete

210. The following are true of management of severe malaria, except:


A. Antibiotics might be given
B. 5 mls of 10% dextrose per kg intravenously
C. Blood transfusion where there high parasitemia even if Hb:7g/dl
D. Artemether is the first line for treatment of severe malaria in Zambia

211. Which of the following is not a complication of severe malaria:


A. Metabolic acidosis
B. Pulmonary hypertension
C. Severe normocytic anemia
D. Bleeding disorders
212. The following statements are true concerning diabetes mellitus type I, except:
A. Somogyi effect leads to low blood sugar levels.
B. Glycosylated haemoglobin helps in the monitoring of management.
C. Diabetic keto-acidosis may be the first presentation in children
D. Enuresis should be alluded to when investigating diabetes mellitus

213. The following is true about child’s development, except for:


A. Cooing and rolling in bed by 3 months
B. Recognizing their own name by 8 months
C. Transferring objects from one hand to the other at 9 months
D. Pincer grasp by 6 months

214. A neonate is born at 34 weeks of gestation. He suddenly becomes pale and


suffers a tonic clonic fit. His motor development is subsequently affected. On
examination, you are likely to find:
A. Spasticity in all four limbs
B. Purposeless limb movements
C. Loss of body balance
D. Jaundice
MK

215. The following statements related to cerebral palsy in children are true, except:
A. Some have a degree of learning impairment
B. Birth asphyxia is the most common cause
C. The ataxic type is the most common clinical type
D. Management should be centered on the clinical type

216. The following are known causes of Cerebral Palsy, except for:
A. Severe untreated neonatal jaundice
B. Hyperglycaemia
C. Periventricular leukomalacia
D. Twin-to-twin transfusion

217. There are three common forms of spina bifida: ______________,


meningocele, and myelomeningocele
A. Spina bifida major
B. Spina bifida occulta
C. Spina bifida minor
D. Anencephaly

218. When considering underlying possibilities for a bleeding tendency (such as


platelets or proteins), one must keep in mind categories of abnormality, such as:
A. Quantity
B. Quality
C. Both
D. Neither

219. The reason subjects with cystic fibrosis have abnormal clotting is:
A. Because of their chronic hypoxia they are clumsy and injure themselves
more frequently.
B. They have trouble absorbing fats, and by extension fat soluble vitamins
C. Their excessive infections make them prone to DIC.
D. This CF gene is closely linked to the factor VIII gene, and they are likely
to inherit both.

220. The most common manifestation of rheumatic fever is:


A. Carditis
B. Arthritis
C. Chorea
D. Subcutaneous nodules
MK

221. All of the following are manifestations of arthritis of Rheumatic Fever,


except:
A. Most commonly affect the knee
B. Characterized by swelling disproportionate to pain
C. Swelling in an individual joint last for 4 to 7 days
D. No residual deformity

222. Concerning Rheumatic Fever, the following are common manifestations of


carditis, except:
A. Cardiomegaly
B. Cardiac failure
C. Cardiac tamponade
D. None

223. All of the following are true about idiopathic nephrotic syndrome, except:
A. Most commonly appears between the ages of 2 and 6 years
B. More common in females than males
C. MCNS is present in 85-90 % of patients of less than 6 years of age
D. FSGS develops in older children

224. The following are uncommon findings in a patient with idiopathic nephrotic
syndrome, except:
A. Abdominal pain
B. Gross hematuria
C. Anorexia and Irritability
D. Hypertension

225. One of the following malaria types is known to cause nephrotic syndrome:
A. Plasmodium falciparum
B. Plasmodium vivax
C. Plasmodium ovale
D. Plasmodium malariae

226. The following are not true of immunization in children, except:


A. Polio and Measles are live vaccines
B. OPV4 has no place in the Zambian calendar
C. Live vaccines, such as measles, are contraindicated in HIV disease
D. Immune response to one vaccine interferes with other vaccines
MK

227. The following statements about BCG vaccine are false, except:
A. It is a killed vaccine
B. It is recommended to be given at 6 weeks of age
C. Is given through a subcutaneous injection
D. It provides more consistent protection against TB meningitis than
pulmonary disease

228. Most deaths due to acute diarrhoea result from the following, except:
A. Associated shock
B. Dehydration
C. Electrolyte imbalance
D. Rotavirus infection

229. The following are biological factors that increase susceptibility to acute
diarrhoea, except:
A. Failure to get immunized against rotavirus
B. Failure of measles vaccination
C. From birth to six months
D. Malnutrition

230. The following are true of fluid management in children, except:


A. Physiology in neonates is the same as in older children
B. Deficit is a 11 kg is estimated at 1100 mls
C. Insensitive water losses in preterm is greater in preterm infants
D. In moderate dehydration, the deficit is estimated at 5 to 10% of the body
weight

231. Possible causes of failure to thrive are listed below, except:


A. Tetralogy of Fallot
B. Bronchiectasis
C. Giardiasis
D. Crohn’s disease
MK

232. All of the following are transmitted as congenital infections, except:


A. Hepatitis A
B. Herpes
C. Toxoplasmosis
D. Syphilis

233. The following clinical features are highly associated with congenital
infections, except:
A. IUGR infants
B. Hepatosplenomegaly
C. Unusual rash
D. Fever

234. The following statements are true of congenital infections, except:


A. The majority of them are asymptomatic at birth
B. Chorioretinitis features in a number of them
C. Lymphadenopathy is common than hepatosplenomegaly
D. Thrombocytopenia is highly suggestive of the diagnosis

235. The following statements concerning cephalohematoma are true, except:


A. Does not cross sutures
B. Fluctuation is not present
C. Present at birth
D. Presents few days after birth

236. The characteristics of caput succedaneum include all, except:


A. Crosses midline
B. Crosses suture line
C. Takes weeks to resolve
D. It is diffuse oedematous swelling of soft tissue of the scalp

237. Of the following, which is not a cyanotic congenital heart lesion?


A. Transposition of great arteries
B. Tetralogy of Fallot
C. Triscupid atresia
D. Atrial septal defect
MK

238. Mutale, a 18 month old toddler, is brought to the follow-up clinic 3 months
after suffering from meningitis. Her fronto-occipital circumference is said to be
expected for age. Choose the most likely correct measurement:
A. 41 cm
B. 38 cm
C. 45 cm
D. 49 cm

239. The most developed sense in neonates is:


A. Smell
B. Touch
C. Hearing
D. Sight

240. Sabina is able to sit without support and cries in the hands of strangers. Her
mother is excited to see her transferring toys from one hand to the other. Her
average age is:
A. 5 months
B. 7 months
C. 9 months
D. 12 months

241. A small-for-date neonate presents with hepatosplenomegaly, the following are


possible diagnoses, except:
A. Syphilis
B. Malaria
C. Rubella
D. HIV

242. The following are suggestive facts about HIV infection in a newborn, except:
A. Prematurity
B. Intra-uterine growth restriction
C. Hepatomegaly
D. Generalised lymphadenopathy
MK

243. The following conditions are stage II, except:


A. Diarrhoea
B. Extensive tinea capitis
C. Dental caries
D. Otitis media

244. The disease staging in HIV is important because the following reasons,
except:
A. It provides guidance to disease prognosis
B. It helps with monitoring response to therapy
C. It helps with the choice of antiretroviral drugs
D. It determines response to therapy

245. The following are manifestations of extra-pulmonary tuberculosis, except:


A. Meningitis not responding to antibiotic treatment
B. Pericardial effusion
C. Painful enlarged joint
D. Distended abdomen with ascites

246. The following are benefits of breastfeeding, except:


A. Provides superior nutrition for optimum growth
B. Provides more iron than any other food for brain growth
C. Provides adequate water for hydration
D. Protects against infection and allergies

247. The following statements concerning cephalhaematoma are true, except:


A. Does not cross sutures
B. Fluctuation is not present
C. Is due to an ischaemic process
D. Presents few days after birth

248. The characteristics of caput succedaneum include all, except:


A. Crosses midline
B. Crosses suture line
C. Easily gets infected if not well managed
D. It is diffuse oedematous swelling of soft tissue of the scalp
MK

249. The following findings are common in Sickle Cell Disease, except:
A. Jaundice is not necessarily pronounced in sequestration crisis
B. Clubbing is not a common finding
C. Acute chest syndrome is often associated with sepsis
D. Splenic sequestration crisis occurs mainly after the age of 5 years

250. The following are true about haematologic changes in Sickle Cell Disease,
except:
A. MCV is always elevated due to folate deficiency
B. Target cells are commonly found on the peripheral blood smear
C. Iron deficiency may occur despite re-use after hemolysis
D. Red blood cell distribution width (RDW) is usually increased

251. The following are common manifestations of carditis in rheumatic heart


disease, except:
A. Anemia is very common
B. Splenomegaly is a common feature
C. Urinalysis may display blood
D. Mitral valve is the less affected

252. Concerning screening of congenital infections, the following statements are


true, except:
A. Ig A in the neonate is evidence of infection
B. Ig M does not cross the placenta and is an indication of active infection in
the neonate
C. Maternal Ig G are passively passed to the neonate
D. Ig G does not cross the placenta and is a sign of active infection in the
neonate

253. In gestational diabetes mellitus, the following are associated causes of


cyanosis in the neonate, except:
A. Transient Tachypnoea of the Neonate
B. Respiratory Distress Syndrome
C. Hyperglycaemia
D. Hypoglycaemia
MK

254. In relation with labour associated cyanosis in the neonate, the following are
correctly matched, except:
A. Prolonged rupture of membranes………..Sepsis
B. Caesarian section…………………….Transient Tachypnoea of the Neonate
C. Prematurity…………………………..Meconium aspiration
D. Sedatives…………………………….Apnoea

255. These hematological changes are characteristic of neonatal sepsis, except:


A. Thrombocytopenia
B. Mandatory high White Blood Cell count
C. Anaemia
D. Elevated C Reactive Protein is diagnostic

256. Cerebral palsy can be best defined as:


A. Stiffness of all four limbs in an infant as a result of an insult on the fetal or
infant brain
B. Permanent disorder of movement and posture due to non –progressive insult
on a developing infant brain
C. Limitation in the range of limbs movement caused by an infectious process
on the fetal or infant brain.
D. A condition characterized mainly by mental retardation as a result of an
insult on a developing fetal or infant brain

257. Which of the following occurrences is not common in normal neonates**


A. Breast engorgement
B. Jaundice
C. Vaginal bleeding
D. None
MK

SECTION 2: TRUE/FALSE
1. Complications of Measles include
A. Xeropthalmia- True
B. Laryngo-tracheo-bronchitis- True
C. Acute Encephalitis- True
D. Giant cell pneumonia- True

2. Concerning Bronchiolitis
A. The commonest causative organism is parainfluenza virus-False
B. The commonest causative organism is Respiratory Syncytial Virus-True
C. The cough and wheeze are due to blockage of the small airways due to
secretions and inflammation- True
D. The cough and wheeze are due to bronchospasm of the bronchi and
bronchioles-False

3. The following are true of Respiratory Distress Syndrome:


A. It is very much dependent on the lecithin-sphingomyelin ration in the
surfactant- True
B. Infants of diabetic mothers are exempted because they are born big even when
they are preterm-False
C. Small for gestational age infants are less likely to get it-True
D. It presents with respiratory distress within 4 hours of birth and wanes within
24 hours-False

4. The following are immediate complications of Birth Asphyxia:


A. Acute tubular necrosis-True
B. Cardiomyopathy-False
C. Cerebral palsy-False
D. Hypoxic Ischaemic encephalopathy-True

5. Concerning Ventricular Septal Defect:


A. It is an acyanotic congenital heart disease-True
B. It is characterised by a pansystolic left sternal border murmur-True
C. It is characterized by a pan-systolic murmur in the mitral area-False
D. It is common in Trisomy 21 and Turner syndrome-False
MK

6. The following are Jones ‘minor criteria in Rheumatic Fever:


A. Huntington’s chorea-False
B. Arthritis-False
C. Elevated C reactive protein-True
D. Arthralgia-True

7. Subacute Bacterial Endocarditis occurs in the following conditions:


A. Ventricular Septal defect-False
B. Rheumatic Heart Disease-True
C. Rheumatic fever-True
D. Tetralogy of Fallot-True

8. Cyanotic congenital Heart Disease include the following:


A. Transposition of Great Arteries-True
B. Patent Ductus Arteriosus-False
C. Tetralogy of Fallot-True
D. Eisenmenger complex- True

9. In children, causes of a prolonged fever include the following


A. Tuberculosis-True
B. Lymphoma-True
C. Juvenile idiopathic arthritis-True
D. Drugs-True

10.The following are features of Down’s syndrome:


A. Down-slanting palpebral fissures-False
B. Hypotonia- True
C. Mental retardation-True
D. Microcephaly- False

11.Prolonged jaundice in a neonate can be caused by:


A. Hypothyroidism-True
B. Galactosaemia-True
C. Neonatal sepsis-False
D. ABO incompatibility-False
MK

12.Recognised consequences of Rhesus incompatibility include:


A. Jaundice on the first day of life-True
B. Positive Apt test-False
C. Increased severity in further pregnancies-True
D. Normal haemoglobin on the first day of life-False

13.Hypertension in childhood typically complicates


A. Post-streptococcal glomerulonephritis- True
B. Coarctation of the aorta-True
C. Minimal change nephrotic syndrome-False
D. Appendicitis-False

14.Causes of unconjugated hyperbilirubinaemia include the following


A. Biliary atresia-False
B. Physiological jaundice-False
C. Galactosaemia-False
D. ABO incompatibility-True

15.The following features suggest a diagnosis of congenital infection in a newborn


infant:
A. Growth retardation-True
B. Petechiae-True
C. Microcephaly-True
D. Intracranial calcification-True

16.In suspected acute rheumatic fever the following indicate rheumatic fever:
A. An erythematous serpiginous rash on extensor surfaces of the patient’s body-
True
B. Clumsiness and unusual dropping of things including a change in
handwriting-True
C. Shortened PR interval-True
D. Hyperpigmented nodules on the extensor surface of the limbs-False
MK

17.Human milk contains more of the following than cows' milk formula:
A. Lactalbumin-True
B. Calcium- False
C. Iron-False
D. Immunoglobulins-True

18.Management of “minimal change “nephrotic syndrome include the following


A. ACE inhibitor-False
B. Anticoagulant-True
C. Antibiotics-True
D. Diuretics-False

19.Concerning post-streptococcal glomerulonephritis


A. It is characterised by high C3 in the acute phase due to the initial
inflammatory process-False
B. It follows infection with Lancefield Group B streptococci infection-True
C. Initial oliguria is succeeded by polyuria as the disease progresses- False
D. Hypertensive encephalopathy is a recognized complication-True

20.Tetralogy of Fallot include the following:


A. A ventricular septal defect with right to left shunt-True
B. The murmur is mainly due to blood flow through the VSD-False
C. Persistence of arterial canal is life-saving- True
D. Plain chest X-ray shows plethoric lung fields-False

21.The following are typical features of sickle cell anaemia


A. An obvious splenomegaly at the age of 10 years-False
B. Bossing due to multiple avascular necrosis in the cranial bones-False
C. Cholelithiasis occurs as early as two years old-True
D. Mental retardation-False

22.Sickle cell patients are prone to infections with these organisms:


A. E. coli-True
B. Haemophilus influenza-True
C. Streptococcus pneumonia-True
D. Measles virus-False
MK

23.Good prognostic signs in acute lymphoblastic leukaemia include


A. Less than 2 years at presentation-True
B. Male sex- False
C. A white cell count above 50,000 but below 100,000-False
D. Mediastinal widening in chest X ray-False

24.The following are common findings in HIV infected neonates:


A. Prematurity-True
B. Parotid enlargement-False
C. Intra-uterine growth restriction-True
D. Hepatomegaly-True

25.The following conditions are WHO stage III:


A. Extensive chicken pox-False
B. Oral candidiasis-True
C. Extensive dental caries-False
D. Oral hairy leucoplakia-True

26.The following are possible features of a rapidly progressive HIV disease in an


infant:
A. Generalised lymphadenopathy-False
B. Oral thrush during the neonatal period-True
C. Hepatosplenomegaly-True
D. Molluscum contagiosum-False

27.Causes of acute inspiratory stridor in children include:


A. Laryngo-tracheo-bronchitis-True
B. Laryngeal papilloma-True
C. Retro-pharyngeal abscess-False
D. Acute Bronchiolitis-False

28.Causes of finger clubbing include:


A. Bronchiolitis-False
B. Pneumocystis pneumonia-False
C. Ulcerative colitis-True
D. Bronchiectasis-True
MK

29.Symptoms and signs of retinoblastoma include the following:


A. A painful red eye-True
B. A new squint or strabismus-True
C. Corneal opacity- False
D. Leukocoria-True

30.The following are common characteristic features of acute glomerulonephritis:


A. Oedema-True
B. Fits-True
C. Normal to nephrotic range proteinuria-True
D. Macroscopic or microscopic haematuria-True

31.Features diagnostic of kwashiorkor include the following


A. Weight 60-80% of expected weight for age with oedema-True
B. Weight >60% of expected weight for age with oedema-False
C. Mental retardation-False
D. Muscle wasting-True

32.The initial management of oedematous malnutrition include the following:


A. Diuretics if oedema is massive-False
B. Antibiotics even with no evidence of sepsis-True
C. Room temperature of about 30 C-True
D. Combined Minerals and Vitamins including copper and selenium-True

33.The following are characteristics of meningitis in children:


A. Naso-pharyngeal bacterial carriage is often the source of infection-True
B. A traumatic lumbar puncture is characterized by CSF ratio of 1
WBC/100RBCs-False
C. Meningococcemia can manifest without meningeal signs-True
D. Gram negative organisms are less common in infants-False

34.Lumbar puncture is contra indicated in the following circumstances:


A. One fit in 24 hours-False
B. Bulging fontanel-False
C. Low pulse rate associated with high blood pressure-True
D. Anisocoria-True
MK

35.Differential diagnosis of a child presenting with history of fever for 2 weeks and
splenomegaly include the following:
A. Salmonellosis-False
B. Tropical splenomegaly syndrome-True
C. Rheumatic fever-False
D. Lymphoma-True

36.Imperative measures to treat Diabetic Keto-Acidosis include:


A. Correction of dehydration over 24 hours-False
B. Infusion of Sodium bicarbonate to correct acidosis-False
C. Infusion of 40 mmol of Potassium chloride-True
D. Empirical administration of antibiotics-True

37.Concerning childhood urinary tract infection:


A. It is commonest in girls under one month of age-False
B. It may present with nausea and vomiting-True
C. It is usually caused by E.Coli-True
D. It may be indicated by a positive nitrite test on urinalysis-True

38.Burkitts lymphoma:
A. Is a tumour commonly found in Africa-True
B. Is a tumour presenting commonly between 4-8 years of age-True
C. Always presents with a mass in the superior mediastinum-False
D. Is highly responsive to chemotherapy-True

39.Regarding childhood malignancies:


A. A mediastinal mass is a frequent finding in T cell Acute Lymphoblastic
Leukaemia-True
B. A bone marrow biopsy should always be performed to exclude leukaemia-
True
C. Cranial irradiation before the age of 3 years has a high neurocognitive
morbidity-True
D. Chemotherapy may predispose to development of secondary malignancies-
True
MK

40.Common manifestations of congenital rubella syndrome include:


A. Cataracts-True
B. Deafness-True
C. Hydrocephalus-False
D. Microcephaly-True

41.A 4-year-old boy is diagnosed with chickenpox. The following would be


characteristics:
A. A maculopapular rash arising from the nape of neck-False
B. A Centripetally distributed rash in crops-False
C. A centrifugally distributed rash comprising papules, pustules, vesicles-True
D. Incubation period of 10 – 21 days-True

42.A 2 days old infant presents with fever and a whimpering cry. These investigation
results would support a diagnosis of neonatal sepsis:
A. Neutropenia-True
B. Leukocytosis of over 25,000/mm3-True
C. Anaemia-True
D. Thrombocytopenia-True

43.The following clinical features are commonly associated with congenital


hypothyroidism:
A. Hypothermia-True
B. Diarrhoea-False
C. Prolonged unconjugated hyperbilirubinaemia-False
D. A small anterior fontanelle-False

44.Febrile convulsions are characterised by the following:


A. Generalised tonic clonic convulsions lasting about 20 minutes-False
B. Very high temperatures may trigger fits in neonates-False
C. The fever has no underlying causative agent-False
D. Occurs not more than once a day-True
MK

45.Which of the following are ways to measure activity level in an infant?


A. How often they feed-True
B. How much they sleep-True
C. How much they want to be held-False
D. All of the above-False

46.The following statements apply to a newborn skull?


A. The skull is made of three separate bones separated by sutures-False
B. The anterior fontanelle usually closes before the posterior fontanelle-False
C. Craniosynostosis is often associated with microcephaly-True
D. Hypothyroidism delays the closure of fontanelles-True

47.Weight gain in the first year life :


A. 750 g / month for the first 4 months-True
B. 200 g / month for the first 3 months-False
C. 250 g / month for the last 4 months-True
D. 500 g / month from the 5th to the 8 th month-True

48.The following are facts about child development:


A. Babbling by 6 months-True
B. Transferring objects from one hand to the other by 4 months-False
C. Does not make eye contact at 3 months until 6 months-False
D. Turns head to visually track objects while sitting-True

49.The following statements regarding Somogyi effect as a result of diabetes in


children are correct:
A. It may occur even in the absence of insulin therapy-False
B. Blood sugar levels are high in the morning before breakfast-True
C. Cortisol and catecholamines are known to enhance this phenomenon-False
D. It occurs mainly in poor controlled diabetes-True
MK

50.The following are possible presenting signs and symptoms of diabetes mellitus
Type I in children:
A. Bed-wetting- True
B. Excessive hunger-False
C. Vaginal candidiasis-True
D. Persistent abdominal pain-True
MK

SECTON 3: ESSAYS
1. Kasongo, a 5-year-old boy of Chingwele, presents with a mandibular mass
causing dental malalignment. Histopathology results reveals a “starry sky”
appearance. (15 marks)
A. What is the most likely diagnosis?
Answer: Burkitt’s lymphoma

B. Write down investigations you would carry out prior to chemotherapy


Answer:
1. Full blood count
2. Liver function tests
3. Urea and creatinine
4. Cross-match and save
5. Serum electrolytes

C. Mention 2 crucial measures you would take prior to and during chemotherapy
Answer:
1. Rehydration
2. Allopurinol administration

D. Write down the most common complication related to chemotherapy of this


condition.
Answer: Tumor lysis syndrome

E. Give at least 3 biochemical derangements associated with this complication


Answer:
1. Hypocalcemia
2. Hyperuricemia
3. Hyperkalemia
4. Hyperphosphatemia
MK

2. A 4-month-old female infant is brought to OPD for not gaining weight. There is
a grade III pansystolic murmur at the left lower sternal border with loud 2nd heart
sound at pulmonary area. Echocardiography reveals Tetralogy of Fallot. (15
marks)
A. Mention at least 4 clinical features compatible with this condition
Answer:
1. Failure to thrive
2. Excessive sweating when feeding
3. Finger clubbing
4. Hyper-cyanotic spells
5. Polycythemia

B. Write down expected hematological (hemogram) findings


Answer:
1. Polycythemia
2. Thrombocytopenia

C. List down expected chest radiological findings


Answer:
1. Boot shaped heart silhouette
3. Olighaemic lung fields: dark lung fields with absent lung markings

D. List 3 complications of this condition.


Answer:
1. Thrombo-embolic accident
2. Cerebral abscess
3. Heart failure
4. Pulmonary regurgitation
5. Arrhythmias

E. Write down 4 drugs (each coupled with its purpose) that are usually used in
this condition
Answer
1. Beta blocker (propranolol): used to prevent cyanotic spells, they reduce
ventricular infundibular spasms
2. Analgesics (Morphine): used to relax the pulmonary infundibulum, reduce
ventilatory drive and for sedation
MK

3. Prostaglandins e.g. alprostadil: Prostaglandin E is used to keep the ductus


arteriosum open enabling blood to shunt back from the left to the right
4. Sodium bicarbonate: corrects acidosis and eliminates the respiratory center
stimulating effect of acidosis
5. Alpha adrenergic agonist (phenylephrine): to increase systemic vascular
resistance reducing shunting of blood from the right side to the left
6. Ketamine: increase systemic vascular resistance and sedates the patient

3. Rupende, a 10-year-old boy is brought into the emergency room with history of
difficulty breathing since the previous night. He is a known sickle cell disease
patient who has had several vaso-occlusive crises monthly. (15 marks)
A. Write down 2 ways you would manage this patient besides the usual IV fluid
and analgesics
Answer:
1. Monthly top up blood transfusion
2. Hydroxyurea

B. During one of the painful crises, he presents with fever and respiratory
embarrassment. Give 2 possible differentials
Answer:
1. Bacterial pneumonia
2. Acute chest syndrome

C. Write down 2 possible causative organisms of the above differentials


Answer:
1. Streptococcus pneumoniae
2. Mycoplasma pneumoniae

D. Mention 2 groups of antibiotics you would give to manage your differentials


Answer:
1. Penicillins
2. Macrolides
MK

4. Itumeleng, a 3-week-old healthy male neonate, is thriving well. However, the


mother notices that he throws up shortly after feeds and very much eager to feed
despite this occurrence. The vomitus is made of the just ingested milk. His blood
count is within normal range apart from a mild anemia. He is not opening bowels
as frequently as he used to. (15 marks)
A. What is your differential diagnosis?
Answer:
1. Hypertrophic pyloric stenosis
2. Antral or pyloric web
3. Trachesophageal fistula
4. Duodenal atresia

B. How do you confirm the diagnosis clinically?


Answer: Test feed reveals an olive shaped mass in the right upper quadrant of
the abdomen. Visible peristalsis may be seen.

C. List down 4 biochemistry expected in this condition


Answer:
1. Hypokalemia
2. Hypochloremia
3. Metabolic alkalosis
4. Elevated blood urea nitrogen

D. What gender does this condition occur mostly in?


Answer: Male
MK

5. A 3-day old infant presents with jaundice which started on the first day after birth
and involved the abdomen below the umbilicus to the knees. He looks lethargic,
irritable and pale. His body temperature is normal. He is the younger of two
siblings. Kernicterus is entertained as the diagnosis (15 marks)
A. What is the Kramer grade of this neonate and its corresponding serum
bilirubin levels?
Answer: Kramer Grade 3

B. Grade kernicterus in this infant


Answer: Moderate kernicterus

C. Direct Coombs test is performed and turns out positive. Whose blood, do you
think, was tested?
Answer: The neonate

D. You decide to institute phototherapy. List down at least 4 measures you would
take prior to and while the baby is undergoing it.
Answer:
1. Maintain room temperature at about 30oC
2. Check blood sugar at regular intervals
3. Encourage breastfeeding
4. Give maintenance fluid with 10% to cover for the deficit

E. Write down 2 complications related to phototherapy


Answer:
1. Dehydration
2. Bronze baby: especially when phototherapy is used for conjugated
hyperbilirubinemia
3. Diarrhea
MK

6. Olipa, a 4-year-old toddler, has recently moved with her parents into the newly-
bult farm house. She is brought to the Emergency Department and with difficult
breathing associated with foaming in the mouth. The father denies any history of
body hotness and fits.
A. Outline your management of this child in 4 bullet points on arrival
Answer:
➢ ABCs: Suction secretion, intubate and give oxygen, gain venous access and
start IV fluids
➢ Stop exposure: remove clothes, wash the child and have her put on different
clothes
➢ Administer activated charcoal for gastric decontamination
➢ Labs: urinalysis, Arterial blood gasses, Full blood count, urea, electrolytes and
creatinine, liver function tests, liver enzymes, serum glucose and RBC
cholinesterase level

B. What is your most likely diagnosis?


Answer: Organophosphate poisoning

C. Write down 4 clinical findings that would help you arrive at the diagnosis?
Answer:
➢ Miosis
➢ Wheezing
➢ Muscle twitching and fasciculations
➢ Lacrimation and diaphoresis
➢ Bradycardia

D. What is the drug of choice in this condition?


Answer: Atropine (0.05mg IV)

E. When would you discontinue the administration of this drug?


Answer: When the patient is atropinized (increase in heart rate, skin is warm
and appears flushed and pupils dilate)
MK

7. Maureen, a 3-year-old girl woke up this morning and is failing to stand. Her
mother thought she is deliberately feigning to attract attention. She however
decides to take her to the hospital. A quick assessment reveals Maureen has
indeed lost pain sensation in both lower limbs up to the knees.
A. What is your working diagnosis?
Answer: Acute flaccid paralysis

B. What is your differential diagnosis?


Answer:
➢ Guillain Barre syndrome
➢ Poliomyelitis
➢ Transverse myelitis
➢ Tick paralysis

C. What is your most likely diagnosis?


Answer: Guillain Barre syndrome

D. What clues would you be looking for in the history as a lead to the diagnosis?
Answer:
➢ History of respiratory illness, pharyngitis or Gastroenteritis (diarrhea and
vomiting)
➢ Presence of fever, generalized non-throbbing headache, sore throat,
anorexia, nausea, vomiting, muscle aches
➢ Bladder (urinary retention) and bowel complaints: absent or present
➢ Immunization history against polio and any recent vaccines given to the
child
E. Give 2 crucial investigations that would help confirm the diagnosis?
Answer:
➢ Nerve conduction test
➢ Lumbar puncture for CSF analysis

F. What’s your worry about this child?


Answer: Respiratory failure or paralysis of respiratory muscles as paralysis
ascends with disease progression
MK

8. Mweetwa is born to a mother living with HIV. He weighs 3.1kg at birth and is
exclusively breastfed. The mother started taking antiretroviral drugs (ARVs) 8
months into her pregnancy.
A. Write down the prophylactic ARVs regimen you would use on Mweetwa?
Answer: AZT + 3TC + NVP

B. Mention 3 advices you would give the mother to ensure she does not pass the
infection to the baby?
Answer:
➢ Exclusive breastfeeding
➢ Mother should continue taking ARVs
➢ Administer ARV prophylaxis and cotrimoxazole prophylaxis to the child
as long as child is exposed

C. How long would you advise the mother to breastfeed the baby?
Answer: as long as she wants as long as she is taking her ARVs rigorously
however WHO advocates for 6 months exclusive breast feeding

D. In a succinct manner, write down the timeline of the HIV testing and the type
of test at every milestone as part of follow-up of Mweetwa
Answer:
➢ Nuclei acid test (NAT) at first contact
➢ NAT at 6 weeks
➢ NAT at 3 months
➢ NAT at 6 months
➢ NAT at 9 months
➢ Serological test (Rapid test) at 12 months (confirm positive with NAT)
➢ Serological test (Rapid test) at 18 months (confirm positive with NAT)
➢ Serological test at 24 months (confirm positive with NAT)
➢ Serological test 6 weeks after complete cessation of breastfeeding

E. It turns out that Mweetwa’s NAT result is positive. What ARV regimen would
you start Mweetwa on?
Answer: AZT + 3TC + LPV-r
MK

9. Chabala, a 3-year-old toddler, is admitted with a 5 day long febrile illness. The
mother reports that all started with body hotness associated with headache. He
was then taken to the local clinic where the health worker commenced him on
penicillin V for 5 days. His condition slightly improved, however a day prior to
admission, he had a fit which prompted the mother to take him to the hospital.
He is HIV negative

On examination, he is somehow irritable and displays a subtle squint. He weighs


11.4 kg. With an axillary temperature of 37.9oC
A. Write down 3 possible diagnoses
Answer:
➢ Cerebral abscess
➢ Meningoencephalitis
➢ Severe malaria

B. Write down one procedure you would avoid due to physical findings
Answer: Lumbar puncture

C. Write 3 important investigations you would carry out besides FBC to arrive
at the diagnosis
Answer:
➢ CT scan of the head
➢ Blood culture
➢ RDT for malaria

D. Outline in 3 bullet points your management besides ABC


Answer:
➢ Continue intravenous antibiotics: Crystalline penicillin 100,000 IU/Kg
QID + IV cefotaxime 100mg/kg QID for 10-14 days as culture results are
awaited
➢ Anticonvulsants and Antipyretic: paracetamol IV
➢ Treat increased intracranial pressure: head elevation (30o)
MK

10.Melanie, a Grade 11 adolescent, attends the OPD complaining of a recurrent


genital thrush which she claimed would clear after treatment, but would resurface
few weeks later. This has been going on for the past 4 months. She is HIV
negative and feeling increasingly weak lately, and has lost weight
A. Write down 2 conditions you would explore during your history taking
Answer:
➢ Diabetes mellitus

B. As you work out the case, you are availed the following results:
WBC: 13,000/mm3, Hb 9.5g/dl, MCV: 75.9fl, RBS: 9.4mmol/L, AST: 39
IU/L, Total serum bilirubin 51 micromol/L
What is your diagnosis?
Answer:

C. Write down one crucial investigation you would carry out to explain what is
happening to Melanie
Answer:

D. Outline your management in 4 bullet points


Answer:
MK

11.Besa, an 18-month-old toddler, lives with her parents who have just moved in
their newly-built house. She is brought to the emergency department with
difficult breathing associated with foaming in the mouth. The father denies any
history of body hotness and fits. She is reported to have been fine the day prior
to the consultation.
A. Outline your management of this child in 4 bullet points on arrival
Answer:
➢ ABCs:
- Airway management: Admit and Keep airway patent
- Suction foam and secretions from the mouth
- Consider intubation
- Keep head end of the bed elevated
- Administer oxygen via nasal prongs or mask
- Gain venous access, send blood for investigations (arterial blood
gasses, U and Es, FBC and Random blood glucose)
- IV fluids (DNS)
➢ Nebulize with salbutamol 5mg
➢ Give hydrocortisone
➢ Antibiotics
B. What is your most likely diagnosis?
C. Answer: Asthma (acute exacerbation)

D. List down 4 possible clinical signs/symptoms that would help with the
diagnosis of this condition
Answer:
➢ Use of accessory muscles of respiration or poor respiratory effort
➢ Presence of wheezing
➢ Impaired level of consciousness
➢ Speaking with difficulty or unable to speak
➢ Heart rate >130b/min
➢ Respiratory rate >50b/min

E. Write down 3 drugs that are critical in the management of this condition
Answer:
➢ Nebulized Salbutamol 5mg
➢ Prednisolone 2mg/kg (max 40mg) PO OD for 3 days
➢ Ipratropium bromide 250mcg
MK

12.Josephine, a teenage mother, delivered a male baby at 37 weeks of gestation,


weighing 2.500kg. She was aided by her mother as the labour could not allow to
reach in time the nearest clinic. He has difficult breathing. His oxygen saturation
is 81 % at room air and 89% through nasal prongs. His body temperature is 35.6o
C. (15 marks)
A. Give 4 possible reasons leading the baby’s current condition.
Answer:
1. Hypothermia
2. Hypoglycemia
3. Meconium aspiration
4. Birth asphyxia

B. Mention 5 measures you would take to manage this baby


Answer:
1. Chest X-ray
2. Warm up the baby and the room
3. Check blood sugar
4. Feeds via nasogastric tube
5. Oxygen by Continuous positive airway pressure (CPAP)

13.Banda, a 9-year-old boy, is brought to AMEU with continuous generalized tonic


clonic seizure for the past 20 minutes. On examination, he is afebrile with no
signs of meningeal irritation. He has scars on his legs. When asked later, he said
they were as a result of sores sustained while setting up rat traps. Other systems
are unyielding. (15 marks)
A. Give 2 bedside assessments that would help your diagnosis
Answer:
1. Urinalysis: proteinuria 1-2+, red cells, red cell and granular casts
2. Blood pressure measurement

B. What is your differential diagnosis?


Answer: Hypertensive encephalopathy due to post-streptococcal
glomerulonephritis
MK

C. In the quest to find the cause of this condition, blood culture is requested for
What would the result likely be and why?
Answer:
No organism because this process is a post infectious one

D. How would you manage Band’s case?


Answer:
• Admit patient
• Dietary sodium/salt (1-2g/day), potassium and fluid restriction
• Oral or intravenous diuretics
• Start antihypertensive drugs (ACE inhibitor)
• Treat seizures with anticonvulsant: diazepam 0.3mg/kg IV or 0.5mg/kg
rectal

14.Write down the Zambian EPI schedule of immunization in a table form (vaccine,
schedule and site) (10 marks)
Answer
Vaccine Schedule Site
BCG At birth Left forearm
POLIO At birth, 6, 10, 14 weeks Orally
and 9 months if polio 0
missed

IPV also given at 14 IM


weeks
ROTAVIRUS At 6 weeks and 10 weeks IM
DPT, HIB, HepB, PCV At 6 weeks, 10 weeks IM thighs
and 14 weeks
MEASLES and 9 months and 18 months IM
RUBELLA

At 14 years girls should receive HPV vaccine.


Routine under 5 services:
➢ Growth monitoring monthly
➢ Deworming every 6 months
➢ Vitamin A supplementation very 6 months
MK

15.Mohamed, a 4-month-old infant, started having diarrhoea 3 days prior to


admission. The mother has spotted blood and mucus in the very frequent (once
watery) scanty stools associated with abdominal cramps. (10 marks)
A. Give your differential diagnosis (4 marks)
Answer:
1. Dysentery
2. Intussusception

B. What is your most likely diagnosis?


Answer: Intussusception

C. Being the intern on call, how would you manage Mohamed’s case?
Answer:
1. Nil per oral
2. Antibiotics (metronidazole)
3. IV fluid (not containing potassium) DNS
4. Group and Cross-match

16.Paul was born via spontaneous vaginal delivery, cried at once and was discharged
the very day. However, the mother noted that Paul would not cry for “food” and
spent most of the time sleeping. When carried, he looked weak, yet with no
apparent reason to explain the generalised weakness. His occiput looks flat and
he is said to have opened bowels 3 days after birth. The mother thought this was
due to the umbilical hernia.
A. What is the most likely diagnosis? (5 marks)
Answer: Down’s syndrome or trisomy 21

B. Write down investigations you would carry out to confirm the diagnosis (3
marks
Answer:
1. Karyotype
2. Abdominal X-ray
3. Abdominal ultrasound
4. Serum urea, electrolytes and creatinine
MK

C. What could be the cause of delay in passing meconium? (2 marks)


Answer:
1. Congenital megacolon
2. Cystic fibrosis
3. Imperforate anus
4. Hirschprung’s disease

17.An 18-month-old boy is brought to the ER with fever, mild pallor and inspiratory
stridor. He is saturating at 94% at room air. The doctor on call immediately starts
the child on oxygen, secures an IV access and administers salbutamol
nebulisation.
A. Write down measures taken by this doctor that are not appropriate for this case
Answer:
➢ Salbutamol nebulization- not appropriate in upper respiratory tract
infection.
➢ Administration of oxygen when child is saturating at 94% on air
(saturation is normal).

B. What is your differential diagnosis?


Answer:
➢ Acute epiglottitis
➢ Viral Croup

C. Three hours later, the stridor gets worse as the child is barely able to breathe
and swallow. What is your most likely diagnosis?
Answer: Acute epiglottitis

D. How would you manage this child at this point?


Answer:
➢ Careful suction of oral secretions avoiding to stimulate the throat.
➢ Oxygen by nasal prongs/tracheostomy
➢ Give steroids: Dexamethasone 0.6-1mg/kg IM or orally repeat 8 hourly
for 2-3 days max or prednisolone 2mg/kg/day orally in 2-3 divided doses
➢ Start antibiotics: 3rd generation cephalosporins or penicillins
- Ceftriaxone 80-100mg/kg/day IM in 2 divided doses for 5 days or
- Cefotaxime 100-150 mg/kg/day in 2-4 divided doses for 5 days or
MK

- Ampicillin IV 200mg/kg/day in 3-4 injections change as soon as


possible to oral amoxicillin 100mg/kg/day in 2 or 3 divided doses) to
complete 5 days treatment or
- Chloramphenicol 50-100mg/kg/day IV in 4 divided doses for 5 days,
switch to oral as soon as possible.

18.A younger mother storms your consulting room panicking about a swelling on
her 5-day old baby. She vehemently denies any trauma, to which her husband
agrees. Besides this, the baby is said to be feeding well. He has no fever.
A. Write down your differential diagnosis.
Answer: Caput succedaneum:
➢ Cephalohematoma (subperiosteal hemorrhage) diffuse, edematous often
➢ Caput succedaneum dark swelling of the soft
tissue of the scalp that
B. On further questioning, the mother swears her baby’s head extends across the
was perfectly well at birth. What is your most likely midline and suture lines.
diagnosis? It is seen in infants
Answer: Cephalohematoma delivered from a face
presentation, soft tissue
C. Write 2 hematological parameters you would like to edema of the eyelids and
monitor. face is an equivalent
Answer: phenomenon. CS may be
➢ Serum bilirubin seen after prolonged
➢ Full blood count: Hemoglobin, RBCs (for anemia), labor in full term or
WBC (infection) preterm infants and is
associated with molding
D. D. How would you manage this neonate of the head.
Answer:
Cephalohematoma
➢ Reassure the parents that it will resolve with time.
(subperiosteal
➢ Conservative management and follow up
hemorrhage) does not
- Observation
cross suture lines. A
- Skull X-ray or CT scan if neurological symptoms appear.
linear skull fracture may
- Monitor for jaundice and rising bilirubin levels. Phototherapy may be
rarely be seen. With time
needed if blood accumulation is significant leading to jaundice
it organizes, calcifies and
➢ If significant in size and does not resolve spontaneously aspiration can be
forms an central
done with prophylactic intravenous antibiotics.
depression.
MK

19.A 7-month-old infant is brought to LMUTH with a four-day history of watery


diarrhoea. While being attended to at the local clinic, he was commenced on
metronidazole and Imodium to stop the diarrhoea to no avail. On examination,
he weighs 6.0 kg and his eyes are sunken. When offered 100 mls ORS, He eagerly
drinks the whole lot to the last drop, but vomits soon after.
A. Assess the hydration status of this child?
Answer: Some dehydration (moderate)

B. Why is this child vomiting?


Answer:
Because the child drank the ORS too quickly instead of in small sips

C. Write down 5 points management of this child


Answer:
• Stop the administration of Imodium and metronidazole
• Rehydrate with 75ml/kg (450ml) ORS in 4 hours, give in small sips from a
cup and if the child vomits wait for 10 minutes and continue slower. Monitor
vitals (respiratory rate, pulse rate) and signs of rehydration and once
rehydrated give ORS 10ml/kg per loose stool.
• Encourage feeding
• Continue breast feeding if child still breast feeding
• Zinc sulphate 20mg per day for 10-14 days

20.A 4-month-old boy is brought to OPD because he is not gaining weight. He is


exclusively breastfed and mother tells that he sweats profusely while taking feed.
He was treated for bronchopneumonia at 2 months of age. Mother says she has
an impression that his lips and tongue look dark all the time. On examination his
weight is 4 Kg, His heart rate is 140/min.
A. What is your likely diagnosis?
Answer: Cyanotic congenital heart disease

B. What additional physical findings are expecting on examination?


Answer:
• Finger clubbing
• Cardiac murmur
MK

• Cyanosis
• Single S2 heart sound

C. How will you investigate this child?


Answer:
• Full blood count
• Chest X-ray
• Electrocardiography
• Echocardiography

D. Write down the likely RBC and platelet count


Answer:
• Polycythemia
• Thrombocytopenia

E. Mention 2 conditions this ailment is associated with


Answer:
• Down’s syndrome
• Ebstein’s anomaly
• Turner’s syndrome

21.A 2-year-old boy presents with cyanosis since infancy. On examination he has
finger clubbing and appears small for age
A. What is your differential diagnosis, at least 4
Answer:
➢ Tetralogy of fallot
➢ Tricuspid atresia
➢ Truncus arteriosus
➢ Transposition of great vessel
➢ Total anomalous pulmonary venous return
➢ Univentricular heart
➢ Hypoplastic left heart syndrome

B. What investigations are you going to do?


Answer:
MK

➢ Echo
➢ ECG
➢ Chest x-ray
➢ FBC/DC

C. The above investigations show that the child has tetralogy of fallot, list the
four characteristic features?
Answer:
➢ Right ventricular hypertrophy
➢ Ventricular septal defect
➢ Pulmonary infundibular stenosis
➢ Overriding aorta

22.Albert, a 3-day old neonate, born to a mother who has been antiretroviral therapy
(TDF /XFT/EFV) for the past 3 years. The viral load at her last check-up is
undetectable.
A. How are you going to manage Albert at this point in time?
Answer:
Perform a nucleic acid test on Albert at this contact
➢ if positive start on treatment on AZT + 3TC + LVP-r or ABC + 3TC +
LVP-r.
➢ If negative then place on prophylaxis AZT + 3TC + NVP for at least 12
weeks or duration of exposure.
Follow up testing at 6 weeks, 3 months, 6 months, 9 months, 12 months, 18
months and 24 months.
Encourage breastfeeding

B. What regimen would commence Albert on and for how long?


Answer: AZT + 3TC + NVP for 6 weeks

C. Write down the schedule of follow-up testing up to the time Albert turns 2
years
Answer:
➢ Nuclei acid test (NAT) at first contact
➢ NAT at 6 weeks
MK

➢ NAT at 3 months
➢ NAT at 6 months
➢ NAT at 9 months
➢ Serological test (Rapid test) at 12 months (confirm positive with NAT)
➢ Serological test (Rapid test) at 18 months (confirm positive with NAT)
➢ Serological test at 24 months (confirm positive with NAT)
➢ Serological test 6 weeks after complete cessation of breastfeeding

23.Chansa, a 7-year-old school-going girl child is brought to the Emergency


Department by her parents with a two-day history of being listless. On
examination, the child is drowsy with an extensive non-blanching rash.
A. What advice would you give the parents?
Answer: Chemoprophylaxis for close contacts with ciprofloxacin.

B. What is the most likely diagnosis?


Answer: Meningococcemia

C. Write down investigations you would carry out?


Answer:
• Lumbar puncture for CSF biochemistry, microscopy/culture and
sensitivity
• Blood culture
• Full blood count with differential count
• Serum blood glucose

D. How would you manage this patient?


Answer:
• Admit the patient and isolate
• Check blood glucose and treat hypoglycemia (5ml/kg of 10% dextrose)
• Treat shock with 20ml/kg normal saline
• Administer intravenous antibiotics (3rd generation cephalosporins):
Crystalline penicillin 100, 000IU/Kg QID + IV cefotaxime 100mg/kg QID
or Ceftriaxone 100mg/kg OD for 10 days
• Corticosteroids: dexamethasone 0.15mg/kg/dose QID or 0.4mg/kg BD for
2 days
MK

24.A 5-day old neonate is brought to the clinic with a 3 days history of fever, poor
feeding and crying a lot. On examination the neonate is irritable, has a bulging
anterior fontanelle and is febrile to touch.
A. What is your diagnosis and differential?
Answer:
Diagnosis: Neonatal meningitis
DDx:
➢ Neonatal sepsis
➢ Subarachnoid hemorrhage

B. What relevant history will you elicit from the mother?


Answer:
➢ History of birth trauma
➢ Duration of labour
➢ Prolonged rupture of membranes
➢ Maternal febrile illness during labor/chorioamnionitis
➢ Any twitching or abnormal movements (seizures)

C. What relevant investigations are you going to do?


Answer:
➢ RBS
➢ FBC/DC
➢ CSF microscopy culture and sensitivity + biochemistry
➢ Blood culture, urine culture
➢ Chest X-ray

D. What might be the characteristic findings on the key confirmatory diagnostic


test?
Answer:
➢ CSF is diagnostic
➢ The most likely causes is acute bacterial meningitis and CSF will show
elevated WBC (polymorphs), high protein and low glucose

E. While you are finalizing you assessment you also note the infant making
cycling and boxing movements with a blank gaze. What is happening and how
will you manage the neonate in totality?
MK

Answer:
➢ The neonate is having a convulsion
➢ Check RBS if hypoglycemic give 10% glucose infusion
➢ Phenobarbitone 20mg/kg IV loading dose to run in 20 minutes and
5mg/kg/day maintenance
➢ Broad spectrum antibiotics: benzyl penicillin + cefotaxime

25.An 11-week-old infant is brought to LMUTH with a four-day history of watery


diarrhoea. While being attended to at the local clinic. His under-five card shows
that he had only received BCG, OPV1.
A. What is the crucial question you would want to ask the mother with regards
to this episode of diarrhoea?
Answer: Breastfeeding?

B. Mention the vaccines that this baby has so far missed.


Answer:
• OPV 0 (at birth) and OPV 2 (at 10 weeks)
• Rota vaccine 1 and 2 (at 6 weeks and 10 weeks)
• Diphtheria, tetanus, pertussis, Hemophilus influenza and hepatitis B 1 and
2 (at 6 weeks and 10 weeks)
• Pneumococcal vaccine 1 and 2 (At 6 weeks and 10 weeks)

C. He takes ORS eagerly. What is his likely hydration status?


Answer: Some dehydration (moderate)
MK

26.Mwale is a 4-year-old girl who has been referred because of poor appetite and
loss of weight. She tested HIV positive two weeks ago by rapid test and has just
finished six months of TB treatment for presumed TB adenitis (persistent
generalized lymphadenopathy) with little response. She is 13kg and a height of
97cm. She has extensive oral thrush, finger clubbing and flat wart lesions on the
face. Her hemogram is as follow: WBC: 4,000/mm3, Hb: 7.4 g/dl, Platelet count:
100,000/mm3.
A. Assign a clinical WHO stage to each of clinical features Mwale presented
with.
Answer:
Persistent generalized lymphadenopathy-Stage 1
Flat wart lesions on the face-Stage 2
Extensive Oral thrush- Stage 3
Finger clubbing-Stage 3
Anemia (Low Hemoglobin <8g/dl)-Stage 3

B. Assign an overall WHO stage to Mwale.


Answer: WHO Stage 3

C. Write down the first line ART drugs you would institute in this case
Answer: AZT + 3TC + LPV-r

27.Sakala is a 4-year-old boy who has been referred because of poor appetite, loss
of weight, and overall failure to thrive. He tested HIV positive two weeks ago by
rapid test, has just finished six months of TB treatment for presumed TB adenitis
owing to cervical, axillary and inguinal lymphadenopathy, with little response.
He is 7.8 kg and a height of 85cm. He has extensive oral thrush and flat wart
lesions on the face and neck. He has hepatosplenomegaly.
A. Write down all the clinical features Sakala presented with and assign a WHO
stage to each one of them.
Answer:
Persistent generalized lymphadenopathy (cervical, axillary and inguinal
lymphadenopathy)-Stage 1
Hepatosplenomegaly-stage 2
Flat wart lesions on the face and neck-Stage 2
Extensive Oral thrush-Stage 3
Weight 7.8kg for age-Stage 4
MK

B. Assign an overall WHO stage to Sakala.


Answer: WHO Stage 4

28.Wezi is a 4-year-old boy who has been referred because of poor appetite, loss of
weight, and overall failure to thrive. He tested HIV positive two weeks ago by
rapid test, has just finished six months of TB treatment for presumed TB adenitis
(persistent generalized lymphadenopathy) with little response. He is 10kg and a
height of 85cm. He has extensive oral thrush and flat wart lesions on the face and
neck. He has hepatosplenomegaly.
A. Assign a clinical WHO stage to each of clinical features Wezi presented with.
Answer:
➢ Persistent generalized lymphadenopathy-Stage 1
➢ Flat wart lesions on the face and neck- Stage 2
➢ Hepatosplenomegaly-Stage 2
➢ Extensive oral thrush-Stage 3
➢ Weight 10kg for age- Stage 4

B. Assign an overall WHO stage to Wezi.


Answer: WHO stage 4

C. Write down the first line ART drugs you would institute in this case
Answer: AZT + 3TC + EFV

29.Wezi is a 4-year-old boy who has been referred because of poor appetite, loss of
weight, and overall failure to thrive. He tested HIV positive two weeks ago by
rapid test, has just finished six months of TB treatment for presumed TB adenitis
owing to cervical, axillary and inguinal lymphadenopathy with little response.
He is 7.8kg and a height of 85cm. He has extensive oral thrush and flat wart
lesions on the face and neck. He has hepatosplenomegaly.
A. Assign a clinical WHO stage to each of clinical features Wezi presented with.
Answer:
➢ Persistent generalized lymphadenopathy-Stage 1
➢ Weight 7.8kg for age- Stage 4
➢ Extensive oral thrush-Stage 3
➢ Flat wart lesions on the face and neck- Stage 2
MK

➢ Hepatosplenomegaly-Stage 2

B. Assign an overall WHO stage to Wezi.


Answer: WHO stage 4

C. Write down the first line ART drugs you would institute in this case
Answer: ABC + 3TC + EFV

30.A younger mother storms your consulting room panicking about streaks of blood
of her 5-day old baby diaper. On examination, you ascertain that it is vaginal
bleeding. However, the baby does not in distress whatsoever. The mother quickly
volunteers that she is keeping her 6-year-old nephew.
A. Write down your differential diagnosis.
Answer:
Neonatal hormone withdrawal bleeding-Maternal estrogen
Trauma (sexual assault/defilement)
Non-sexual trauma

B. Taking into account the above history, what is your most likely diagnosis?
Answer: Neonatal hormone withdrawal bleeding-Maternal estrogen

C. How would you manage this baby?


Answer:
• Examine the integrity of the genital
• Assess the intensity of the flow
• Reassure the mother the problem will resolve withing a few days (within
10 days) and she should come back if it is persistent, increasing or if a foul
discharge is present.
• Conservative management: advise the mother to clean the infant’s genitals
with warm water only, spread the labia and gently wash out the creases and
there is no need to go any deeper than this. Wipe from front to back to
avoid contamination of vaginal area from stool.
MK

31.A four-year-old girl presented to the Paediatrics Emergency Services with


extreme pallor, thready pulse associated with a huge spleen. She is almost
unconscious and has no fever. Medical records show that she was been seen a
week before for generalized body weakness and had a tinge of jaundice attributed
to malaria.
A. What is your most likely diagnosis?
Answer:
• Splenic sequestration crises in sickle cell anemia
• Hypersplenism

B. What is the cause of her current state?


Answer: This is due to a sequestration crisis: sickled red blood cells that block
the splenic outflow, leading to the pooling of peripheral blood in the engorged
spleen and features of shock seen due to intravascular hypovolemia and
cardiovascular collapse.

C. Write down steps in managing this emergency


Answer:
• Admit patient
• ABC- Secure airway, ensure patient is breathing consider intubation and
mechanical ventilation, administer supplemental oxygen
• Send blood for sickling test, Full blood count, urgent grouping (X-match
and blood transfusion), Urea, creatinine and electrolytes, Hb
electrophoresis/Peripheral smear
• Start running intravenous fluids as blood is waited for to treat shock
• Blood transfusion
• Cover on antibiotics
• Plan for Splenectomy

32.Mwanja a 35-week-old infant, is admitted with some respiratory distress and


commenced on oxygen 3L/min via nasal prongs and the saturation is stubbornly
stuck at 85% despite increasing the oxygen flow. Mwanja is HIV negative and
his lungs are clear on auscultation. His hematocrit is around 65%
A. Give your differential diagnosis
Answer: Cyanotic congenital heart disease
MK

B. Justify the hematocrit reading


Answer: the hypoxia causes a release of erythropoietin which stimulates
synthesis and release of erythrocytes resulting in polycythemia and a high
hematocrit, additionally, respiratory distress may also cause dehydration with
resultant hemoconcentration also contributing to the rise in hematocrit.

C. Line up the principles of management of this child


Answer:
➢ Medical management
• Administer oxygen
• Place child in Knee-chest position
• Sedation and pain relief with morphine
• Increase systemic vascular resistance and reduce infundibular spasms
and right ventricular outflow obstruction:
- Propranolol
- Phenylephrine
• Alternatively, ketamine can be used to increase systemic vascular
resistance and provide sedation
• Correct acidosis with sodium bicarbonate
➢ Definitive management is by surgery at 6 months of age. In some infants
a surgical placement of an artificial tube between the subclavian and
pulmonary artery (Modified Blalock-Taussig shunt) or sometimes by
dilatation of the right ventricular outflow tract

33.Mweemba, 10-year-old girl is brought into the emergency room with history of
difficulty breathing since the previous night. She is a known sickle cell disease
patient who has been receiving monthly blood transfusions due to cerebral
vascular accident. She is in severe respiratory distress with a respiratory rate of
70/min, associated with subcostal and intercostal recession. Her temperature is
38.7oC, her hemoglobin is 7g/dl and her oxygen saturation is 76%.
A. What are the 2 key differential diagnosis for the condition that the patient is
presenting with?
Answer:
a. Acute chest syndrome
b. Bacterial Pneumonia
MK

B. Besides blood, list 2 further investigations you would carry out


Answer:
a. Chest X-ray
b. CT chest

C. Comment on her hemogram: WBC 22,300 cells/mm3, MCV: 65.7fl, RDW-


CV 25%
Answer:
➢ WBC is raised
➢ MCV is low
➢ RDW is raised

D. How would you mange this child?


Answer:
➢ Admission
➢ Hydration: IV fluids (1.5 x maintenance)
➢ Oxygen support- ventilation (CPAP may be necessary)
➢ Antibiotics: Ceftriaxone and Erythromycin-250mg BD for 10 days
➢ Transfuse packed cells (15-20 ml/kg) slowly over 4 hours and give
1mg/kg Furosemide stat for anemia
➢ Consider partial exchange transfusion
MK

34.A 25-month-old boy is brought to the ER with fever, mild pallor and expiratory
stridor and rhonchi on auscultation. He is saturating at 89% at room air. The
doctor on call decides to immediately intubate and start the child on oxygen. He
also nebulizes the toddler with adrenaline because of the stridor.
A. Write down measures taken by this doctor that are not appropriate for this case
Answer:
• Immediate intubation
• Nebulizing with adrenaline-not appropriate in lower respiratory tract
infection

B. What is your differential diagnosis?


Answer:
• Bronchiolitis
• Asthma

C. The mother then thought it might help to mention that this was not the first
time this is happening. What is your most likely diagnosis?
Answer: Asthmatic attack

D. How would you manage this child at this point?


Answer:
• Admit patient
• Oxygen by nasal prongs
• IV fluids-maintenance
• Nebulization with beta agonist
• Corticosteroid nebulization
MK

35.A 4-year-old boy is brought to the ER with fever, mild bleeding from the gums
and some pallor. He is saturating at 94% at room air. The doctor on call
immediately starts the child on oxygen, orders blood transfusion and administers
Vitamin K. On examination, there is no jaundice and the patient is fairly well
nourished
A. Write down measures taken by this doctor that are not appropriate for this case
Answer:
• Oxygen therapy despite saturating at 94% on room air (normal)
• Administration of Vitamin K
• Blood transfusion when the doctor should have done a Cross-Match and
save

B. Write down 3 prongs approach to the immediate management of this patient.


Answer:
• Run a malaria slide
• Carry out a full blood count/peripheral smear/bone marrow biopsy
• Cross-match and save

C. Give 3 differentials?
Answer:
• Leukemia
• Sepsis and disseminated intravascular coagulation
• Complicated malaria

D. A day later, the haemogram reveals very low white blood cells count. What is
your most likely diagnosis?
Answer: Leukemia

E. How would you manage this child at this point?


Answer:
• Antibiotics
• Blood transfusion
• Chemotherapy
MK

36.Chola a 10-year-old, is admitted with a month-long history of headaches, low


grade fever on and off, he was kept in hospital for few days. 3 weeks ago and
treated for meningoencephalitis and showed some improvement. This time, he is
not able to talk coherently and displays squint. His appetite is poor and he lost
some weight.
a) Give 3 differential diagnoses
Answer:
➢ TB meningitis
➢ Cerebral abscess

b) How would you investigate this boy?


Answer:
➢ Full blood count + Differential count
➢ Blood culture
➢ CT head

c) How would you mange chola?

37.An 11-year-old presents with a nagging itchy skin lesion on the medial malleolus
for 2 months that opened up and keeps growing. Preliminary investigations
reveal:
WBC 13, 000/mm3
Hb g/dl
MCV 75fL
RBS 9.6 mmol/l

a) Give 2 differentials
Answer:
➢ Venous ulcer
➢ Buruli ulcer

b) What is the most likely diagnosis?


Answer: venous ulcer secondary to sickle cell disease

c) List one conclusive test you will do to confirm your diagnosis


Answer: Hemoglobin electrophoresis
MK

d) List 4 bullets on management


Answer:
➢ Pus swab for culture microscopy/culture and sensitivity
➢ IV fluids, crossmatch and transfusion (Hb target >10g/dl)
➢ Antibiotics
➢ Daily wound cleaning

38.A 6-month-old boy presents with cough and fever for 3 days, she was treated for
the cough at the local clinic a week ago and mum says there was not much
improvement. On examination the infant is in respiratory distress, however
feeding okay and is active. On examination the child has bilateral coarse
crepitations and rhonchi with subcoastal recessions
A. What are your diagnosis and differential diagnosis, starting with the most
likely?
Answer:
➢ Acute bronchiolitis
➢ Viral pneumonia
➢ Croup

B. What are the commonest organisms causing the above likely condition?
Answer:
➢ RSV
➢ Influenza virus
➢ Human metapneumovirus
➢ Rhinovirus
➢ Coronaviruses
➢ Parainfluenza virus
➢ Adenovirus

C. List some risk factors for the above condition?


Answer:
➢ Winter months
➢ Cardiac condition
➢ Chronic respiratory condition
➢ Immunosuppression
MK

➢ Age <2 years

D. How will you manage this child?


Answer:
➢ Supportive
- Oxygen
- Antipyretic
- Hydration
- Nutrition

39.A 3kg infant was born to a 16-year-old Para 1 mother. Labor was rather prolonged
lasting 14 hours and there was prolonged rupture of membranes for 24 hours.
APGAR score was 7/10 then 9/10 after some resuscitation. 2 days after birth the
child developed failure to feed, lethargy, jaundice, and hypotonia.
a) What is your differential diagnosis?
Answer:
• Neonatal sepsis
• Birth asphyxia (hypoxic ischemic encephalopathy)

b) What is the most likely diagnosis?


Answer: Early onset Neonatal sepsis

c) List four investigations that must be done in this child to help you confirm the
diagnosis?
Answer:
➢ Full blood count with differential
➢ Blood culture
➢ Total serum bilirubin + direct
➢ Liver function tests
➢ Renal function tests
➢ Blood sugar
➢ Brain imaging (ultrasound, MRI)

d) Write 2 long-term complications associated with your differential diagnosis?


Answer:
• Cerebral palsy
• Learning disability
MK

e) What is your management plan?


Answer:
➢ Admit child
➢ Supportive care
o Insert IV line.
o Provide warmth (maintain temperature 36.5 to 37.5oC).
o Start oxygen by hood or mask 2-3 liters.
o Assess peripheral perfusion (pulses, capillary refill time and skin
color) and Infusion with normal saline or ringer’s lactate 10ml/kg over
5-10 minutes. Repeat the same 1-2 times over the next 30-45 minutes,
if perfusion remains poor start dopamine.
o Treat hypoglycemia by infusing 2ml/kg of 10% dextrose.
o Initial enteral feeds if no abdominal distension and when baby is
hemodynamically stable. Feed with mother’s milk. Consider
parenteral nutrition if baby is not expected to receive enteral feeds for
prolonged period.
o Administer vitamin K 1mg IM if not given.
o Transfuse packed cells if hematocrit (<35-45%).
➢ Antibiotics: Benzyl penicillin (50 000 U/Kg QID IV)/ampicillin (100-
200/kg/day in 4 divided doses) and gentamicin (7.5 mg/kg IV OD or in two
divided doses) for 7 to 10 days as culture results are being waited for,
antibiotics should be tailored to culture and sensitivity reports

40.Emmanuel, a month-old infant, born at term, is admitted for jaundice that


appeared on day 6 after birth and has persisted. He is alert, sucks well and has no
fever. His hemoglobin is 15g/dl with total serum bilirubin of 250 micromol/l.
a) Write down 2 important questions you would ask the mother with regards to
the jaundice
Answer:
➢ What is the color of the stool?
➢ What is the color of the urine?

b) Write down 2 differential diagnoses


Answer:
➢ Breast milk jaundice
➢ Obstructive jaundice (Probably secondary to biliary atresia)
MK

c) Write down your management


Answer:
➢ Management is dependent on the cause
➢ Admit patient
➢ Continue breastfeeding
➢ Investigate: Blood typing/grouping, FBC, Coombs test, total bilirubin, direct
bilirubin, abdominal ultrasound, urinalysis
➢ Hydrate with IV fluids
➢ Consult surgeons if cause is obstructive
➢ For breastmilk jaundice: continue breastfeeding

41.You walk into a mall with your 4-year-old toddler and she picks a pack of food
with the following nutritional information per 100g:
Protein: 10g, Maize 40g, Wheat flour: 50g, Millet flour: 25g, Olive oil: 4g, Palm
oil: 6g, Zinc: 11mg, Potassium 110mg, Vitamin A: 800mcg and folate: 230mcg
a) Work out the energy content of this pack
Answer:
1g protein/carbohydrates= 4kcal
1g fat= 9Kcal

Total protein= 10g


Total carbohydrates= 40g + 50g + 25g= 115g
Total fats= 4g +6g= 10g

Total calories from proteins= 10 x 4= 40kCal


Total calories from carbohydrates= 115 x 4= 460kCal
Total calories from fat= 10 x 9= 90kCal

Total calories in 1 pack= 40 + 460 + 90= 590kCal

Total energy content (in KJ) = 590 x 4.2= 2478Kj

b) Considering that your child’s daily caloric requirement is about 1600kcal,


how many packs would she require to cover this need?
Answer:
If 1 pack gives 590kCal
MK

1600
Then for 1600kCal, number of packs= = 2.711
590
Therefore 3 packs are needed

42.You walk into a mall with your 4-year-old toddler and she picks a pack of food
with the following nutritional information per 100g:
Protein: 12g, Maize 35g, Wheat flour: 70g, Olive oil: 3g, Palm oil: 5g, Zinc:
11mg, Potassium 110mg, Vitamin A: 800mcg and folate: 230mcg
a) Work out the energy content of this pack
Answer:
Answer:
1g protein/carbohydrates= 4kcal
1g fat= 9Kcal

Total protein= 12g


Total carbohydrates= 35g + 70g = 105g
Total fats= 3g +5g= 8g

Total calories from proteins= 12 x 4= 48kCal


Total calories from carbohydrates= 105 x 4= 420kCal
Total calories from fat= 8 x 9= 72kCal

Total calories in 1 pack= 48 + 420 + 72= 540kCal

Total energy content (in KJ) = 540 x 4.2= 2,268Kj

b) Considering that your child’s daily caloric requirement is about 1800kcal,


how many packs would she require to cover this need?
Answer:
If 1 pack gives 540kCal
1800
Then for 1600kCal, number of packs= = 3.33
540
Therefore 4 packs are needed

43.A 13 months old toddler is admitted with acute diarrhea with severe dehydration.
He weighs 10kg.
a) Write down 5 clinical features you are likely to elicit
MK

Answer:
➢ Alteration in consciousness: Lethargic or comatose
➢ Sunken anterior fontanelle
➢ Sunken eyes
➢ Dry mucus membranes
➢ Skin pinch goes back very slowly (>2 sec)
➢ Tachycardia: rapid thready pulse
➢ Oliguria

b) What is the average percentage of his body weight loss?


Answer: more than 10% with severe dehydration

c) Outline the management of this patient


Answer:
➢ Rehydration: WHO plan C
o 100ml/kg Ringer’s lactate or ½ strength Darrows IV
▪ 300ml in the first 30 minutes
▪ 700ml in next 2hrs 30 minutes
o Monitor every 15 minutes to watch out for overhydration (check for
edema, heart rate and respiratory rate) and reassess after 3 hours and
reclassify hydration status
➢ Continue breast feeding if child still breast feeding
➢ Continue feeding
➢ Zinc sulphate 20mg/day for 2 weeks
➢ Educate mother on signs of overhydration and dehydration

44.Chansa, a 6-year-old girl is referred from Kasisi for convulsions. She has had few
episodes of vomiting, fever and weakness. On examination, she is comatose with
a GCS of 8/15, pale and jaundiced
A. Write down 3 possible diagnoses?
Answer:
➢ Severe malaria
➢ Meningoencephalitis
➢ Sickle cell anemia with CVA
➢ Septicemia
MK

B. List down specific investigations to ascertain your diagnoses


Answer:
➢ RDT/MPS
➢ FBC
➢ HB electrophoresis
➢ Cranial CT scan
➢ Peripheral smear
➢ Blood culture

45.Mercy, a 4-month-old infant, is admitted with diarrhoea that has lasted 3 months.
She is HIV exposed and on nevirapine prophylaxis since birth. Her mother has
been on combined ART for the past 3 years. Mercy is on formula milk because
the mother alleged not to have had enough breastmilk. Her father does some piece
work at a nearby mine. On examination, she weighs 3.500 kg with head
circumference of 37 cm. Her birth weight is 2.600 kg.
A. Write down 2 possible causes of the diarrhea in this context
Answer:
➢ Opportunistic infection
➢ Osmotic diarrhea
B. What would you have advised the mother to do about infant feeding in the
immediate postnatal period?
Answer:
➢ Encourage mother to continue HAART
➢ Allow exclusive breastfeeding for 6 months

C. Is her weight appropriate for age?


Answer: No

D. If not, what is her expected weight range for age?


Answer: 4.7-6.2kg

E. List down 2 important investigations in the management of Mercy’s case


Answer:
➢ HIV DNA PCR
➢ Stool microscopy/culture/sensitivity for ova and parasites as well
MK

46.A 5-year-old child is brought to the Emergency Services body hotness for 5 days
associated with generalized body pain. Blood is collected for an urgent
hemogram even before the physical assessment is done and the following is the
result:
WBC: 94,300 /mm3 (Neutrophil: 15%, Lymphocyte: 80.5%), Hb:5.1 g/dl, MCV:
84.7fl, Platelet count: 181,000/mm3, RDW-CV:28.4%.
A. Write down 3 differentials
Answer:
➢ Overwhelming sepsis with leukemoid reaction
➢ Leukemia
➢ Sickle cell disease

B. List down 5 possible clinical findings associated to your differentials


Answer:
➢ Pallor
➢ Hepatosplenomegaly
➢ Delayed capillary refill time
➢ Petechiae
➢ Bone tenderness

C. Give 3 further investigation to confirm each of your differentials


Answer
➢ Sepsis
o Blood culture
o Urine culture
o Lumbar puncture
➢ Leukemia
o Chest X-ray/cytochemistry
o Bone marrow aspiration
o Lumbar puncture/Peripheral smear
➢ Sickle cell disease
o Sickling test
o Hb electrophoresis
MK

o Peripheral smear

47.Ben, a 6-month-old infant is admitted with pneumonia, weighing 4.5kg. His rapid
test for HIV is reactive.
A. Write down 5 questions you would ask the mother to assess the risk of HIV
infection in Ben.
Answer:
➢ Have you tested for HIV?
➢ When did you test for HIV?
➢ Are you on ARVS?
➢ For how long have you been taking the ARVs?
➢ Is Ben currently still exclusively breastfed or given formula?
➢ Is Ben on any prophylactic antiretroviral drugs-Nevirapine at birth?
➢ Was DBS for HIV DNA PCR test done on Ben?

B. How would you manage George at this juncture?


48.
49.George, a 6-month-old infant is admitted with pneumonia (first occurrence) and
is able to feed well. His mother tested positive for HIV.
C. Write down 5 questions you would ask the mother to assess the risk of HIV
infection in George.
Answer:
➢ Are you on ARVS?
➢ For how long have you been taking the ARVs?
➢ What was the mode of delivery: vaginal or cesarean section?
➢ Is George currently still exclusively breastfed or given formula?
➢ Is George on any prophylactic antiretroviral drugs-Nevirapine?
➢ Was Nucleic acid test done on George at 6 weeks?
➢ How has George’s health been and how much does he weigh now?

D. How would you manage George at this juncture?

50.A 14-year-old girl, Nana Maliki presents to your hospital with a history of
breathlessness and swelling of her legs for the past one week. Some of the
physical findings include orthopnea and a pansystolic murmur heard best at the
apical area and radiating to the left axilla.
a) Give your differential diagnosis?
MK

Answer: Congestive heart failure secondary to Rheumatic heart disease


DDx: infective endocarditis

b) Mention two other signs that could be found in this patient consistent with
the above diagnosis?
Answer:
➢ S3 heard sound
➢ Displaced apex heart beat
➢ Bibasal crepitations

c) Line up investigations based on your differential diagnosis.


Answer:
➢ Chest X-ray
➢ Echocardiography
➢ Electrocardiography
➢ Full blood count
➢ Blood culture
➢ Urinalysis

d) Which organisms are usually associated with this condition?


Answer:
➢ Staphylococcus aureus
➢ Streptococcus viridans

d) Which one is the most affected age group and why?


Answer: School going age because of exposure to other possible sources of
transmission
51.An 8-month-old male baby presents to hospital for a third episode in 3 months,
of swelling of the hands and crying uncontrollably. Physical examination reveals
an irritable baby with bilateral swollen hands. Further examination revealed a
tinge of jaundice and splenomegaly.
a) What is your provisional diagnosis in this baby?
Answer: Acute dactylitis in a possible Sickle cell anemia patient

b) What 2 investigations would you request to confirm the diagnosis?


Answer:
➢ Sickling test
MK

➢ Hemoglobin electrophoresis

c) What is the most likely way the child acquired this condition?
Answer: The condition is inherited and the inheritance pattern is autosomal
recessive meaning one can only get the condition if both parents are either
carriers or sufferers of the condition

d) Write 5 complications that can occur in this condition?


Answer:
➢ Cerebrovascular accident: Transient ischemic attacks and stroke
➢ Neurosensory hearing loss
➢ Retinal hemorrhage
➢ Retinal detachment and blindness
➢ Acute chest syndrome
➢ Recurrent pneumonias
➢ Respiratory failure
➢ Pulmonary infarction
➢ Pulmonary hypertension
➢ Myocardial infarction
➢ Arrhythmias
➢ Heart failure and cor-pulmonale
➢ Cholelithiasis
➢ Isosthenuria
➢ Priapism and impotence
➢ Menstrual irregularities, infertility and recurrent abortions
➢ Delayed puberty
➢ Osteomyelitis
➢ Avascular necrosis of head of femur and humerus
➢ Recurrent leg ulcers
➢ Delayed growth development and musculoskeletal deformities

52.A 2-year infant is brought to the Outpatient Department with body hotness. He
weighs 9.9Kg and his haemogram is as follow:
WBC: 25.85 x 103/mm3, RBC: 2.56x106/mm3, Hb: 9.3g/dl, HCT: 31.7%,
RDW-CV: 25%, MCV: 123 fl, Platelets: 376 x103/mm3.
A. Determine the nutritional status of this child using the Welcome
classification.
MK

Answer: 82.3%, Normal nutrition


B. What type of anaeamia is this in view of the other blood indices?
Answer: Megaloblastic anaemia

C. Give 3 possible differentials, based on your findings.


Answer:
Sickle cell anaemia
Sepsis with leukemoid reaction
Leukaemia

C. Give 3 additional laboratory tests you would do to complete your


assessment.
Answer:
Peripheral smear
Reticulocyte count
Bone marrow biopsy
Blood culture
53.
54.A 2-year-old boy presents to 1st level referral hospital where you are working
with history of swelling of the feet for the last one week. There is also history of
diarrhea for 3 weeks prior to admission and a peeling rash on the feet of the child.
The child is the first born. Currently mum is 8 months pregnant. Preliminary
investigation reveals the following:
➢ Potassium: 3.1mmol/l
➢ Sodium 134mmol/l
➢ Chloride 104mmol/l
➢ Blood sugar 1.8mmol/l
a) What is the most likely diagnosis?
Answer: Severe acute malnutrition

b) List 4 diagnostic features that you will look for as you examine the child that
will help you make a more conclusive diagnosis of this condition
Answer:
➢ Decreased mid-upper arm circumference (<11.5 cm)
➢ Weight/height <-3 Standard deviation
➢ Muscle wasting and decreased subcutaneous tissue: prominent outline of ribs
and sternum, baggy pants appearance
MK

➢ Bilateral pedal edema and signs of dehydration (increased capillary refill time
>3 sec, alterations in level of consciousness-drowsiness, lethargy, apathy,
cold peripheries, weak thready rapid pulse, hypotension)
➢ Hair changes: thin, sparse, brittle, brown/reddish discoloration, flag sign
➢ Eye signs: bitot spots, dry conjunctiva, corneal clouding, corneal
ulceration/keratomalacia
➢ Oral changes: cheilosis, angular stomatitis, papilla atrophy on tongue,
abdominal distension, hepatomegaly
➢ Skin changes: desquamation, hypo- and hyperpigmentation, ulceration,
exudative lesions, dryness of skin, crazy pavement and flaky paint dermatosis
➢ Nail changes: brittle spoon shaped

c) Indicate any abnormalities in the above given investigation result


Answer:
➢ Decreased potassium (normal= 3.5 to 5.5 mmol/L)
➢ Low glucose level (hypoglycemia <3 mmol/L)

d) What 5 things affecting this child would likely cause mortality within 24
hours of admission unless taken care on admission?
Answer:
➢ Hypoglycemia
➢ Hypothermia
➢ Dehydration
➢ Electrolyte imbalance
➢ Infection

e) What 5 steps are you going to take immediately to save the life of this child?
Answer:
➢ Manage hypoglycemia
o 5ml/kg of 10% dextrose IV
o Start 2 hourly feeds with F75 (11ml/kg/feed if edema not severe and
8-9 ml/kg/feed if severe)
o Check blood glucose every 30 minutes until glucose level is normal
and stabilizes, if hypoglycemic repeat 5ml/kg 10% dextrose
➢ Manage hypothermia (Axillary temp <35oC or rectal temp <35.5oC, if <32oC
it is considered severe hypothermia)
MK

o Keep the child warm (cloth child, provide heat using overhead warmer,
skin contact and heat convectors)
o Start 2 hourly feeds with F75 (Warm feeds)
o For severe hypothermia
▪ Give humidified oxygen
▪ Give 5ml/kg of 10% dextrose IV
▪ Rewarm child slowly
▪ Start feeds with F-75
o Monitor temperature 30 mins if using a heater otherwise monitor
hourly
➢ Manage dehydration: rehydrate over 12 hours
o If child in shock give 15ml/kg of half-strength Darrows with 5%
dextrose or Ringer’s lactate with 5% over 1 hour and monitor
hydration status (pulse should decrease, respiratory rate should
decrease, capillary refill time should improve and urine should be
passed)
o If still in shock repeat 15ml/kg in next 1 hour.
o After infusion or if child not in shock give 5ml/kg every 30 minutes of
ReSoMal orally or by Nasogastric tube for first 2 hours and then for
next 10 hours alternate ReSoMal with F-75 hourly (same amount
5ml/kg)
➢ Manage electrolyte imbalance (hypokalemia)
o ECG monitoring
o If diarrhea present 600mg OD slow K
o Start potassium chloride infusion at 0.3-0.5 mEq/kg/hour
o Give potassium 4mmol/kg/day for 2 weeks
➢ Treat infection/antibiotics (7 days)
o Uncomplicated: amoxicillin (50-100mg TDS PO)
o Septicemia: IV ampicillin (100-200/kg/day QID) or X-pen (100mg-
200mg/kg/day QID) + Gentamycin (7.5mg/kg OD or in 2 divided
doses)
o If no improvement occurs within 2 days change to IV cefotaxime
(100mg/kg/day TDS) or ceftriaxone (50mg/kg/day BD)

55.A 14-year-old girl called DJ is referred from a health center 10km from the
hospital where you are working with history of east fatiguability, fever and
bleeding from the nose for one month. On examination, you find that the girl is
MK

very pale, has purplish bumps in the mucus membrane of the oral cavity and is
toxic looking. The temperature is 39OC.
a) What 4 critical investigations are you going to do to help you make a definitive
diagnosis on this child?
Answer:
➢ Full blood count with differential: pancytopenia, thrombocytopenia
➢ Peripheral smear to look for blast cells
➢ Clotting profile: aPTT, Bleeding time, prothrombin time for DIC
➢ Chest X-ray to check for any mediastinal masses as well as CT scan
➢ Bone marrow aspirate
➢ Blood culture

b) What are 2 most likely differential diagnosis that would explain the patient’s
condition
Answer:
➢ Idiopathic thrombocytopenic purpura
➢ Leukemia

c) List 6 other signs that you would look for on examination of the patient that
would help you diagnose the patient’s illness.
Answer:
➢ Hepatomegaly
➢ Splenomegaly
➢ Generalized lymphadenopathy
➢ Bone pain
➢ Jaundice, Gum hypertrophy
➢ Cranial nerve palsy, papilledema, meningism
MK

56.A 16-month-old toddler (girl) is brought to the children’s clinic, because the
mother is concerned that the child’s growth and development is suboptimal
compared to her older siblings. Upon further questioning she agrees to the child
having recurrent upper respiratory infections, occasional bouts of loose stool and
fevers that have been treated as “malaria” by the private doctor. On examination
she weighed 7.5kg, her height was 80cm, she had slight pallor with generalized
lymphadenopathy, sores (white patches) in the mouth, CVS was normal and per
abdomen there was hepatosplenomegaly with no pedal edema.
a) Give 4 possible most likely differentials for the described condition.
Answer
➢ HIV infection with TB co-infection
➢ Severe acute malnutrition
➢ Infectious mononucleosis
➢ Enteric fever

b) List 4 investigations that you would do to come up with a definitive diagnosis


of the condition of this child?
Answer:
➢ HIV test
➢ Full blood count with differential
➢ Gastric lavage or sputum for geneXpert
➢ Peripheral smear

c) If you are to stage this condition, what stage of disease is the child in.
Answer: HIV stage 3

57.A 2-year-old girl presents to your hospital and after investigations the CSF
revealed the following results
Appearance: cloudy
WBC: 120cells/mm3- polymorphs 80% and lymphocytes 20%
RBC: 50cells/mm3
Gram stain: gram negative coccobacilli
Ziehl Neelsen stain: no organism
Protein: 0.75g/l
MK

Sugar: 1.6mmols/l
a) List 4 cardinal abnormalities that you see in this CSF result
Answer:
➢ Pleocytosis with predominance of neutrophils but also presence of
lymphocytes is also an abnormality
➢ Presence of gram-negative coccobacilli
➢ Increased proteins
➢ Low blood sugar

b) What is the causative organism?


Answer: Hemophilus influenza type B

c) What specific measure would you have taken to prevent this illness in the
child?
Answer: immunization against Hemophilus infleunza at 6 weeks, 10 weeks
and 14 weeks

d) List 4 long term complications associated with this condition?


Answer:
➢ Hearing loss
➢ Learning disability/mental retardation
➢ Cranial nerve palsy
➢ Focal neurological deficits

58.This is a Hemogram of a patient you have just seen at your hospital:


WBC: 5.7 x 109/L
RBC: 2.8 x 1012/L
Hb: 5.3g/dl
MCV: 49fl
MCH: 18.3pg
Platelets: 272 x 109/L
a) List 4 abnormalities that you see in this Hemogram
Answer:
MK

➢ Low red blood cell count


➢ Low hemoglobin
➢ Low mean corpuscular volume
➢ Low mean hemoglobin concentration

b) What is the diagnosis based on the Hemogram?


Answer: Microcytic hypochromic anemia

c) Give 3 possible causes of this condition.


Answer:
➢ Iron deficiency anemia
➢ Sideroblastic anemia
➢ Thalassemia

59.An 8 weeks old infant presents to your admission ward with history of cough for
4 days and difficult breathing. On examination, the infant is well nourished and
weighs 6kg. However, the infant is dyspneic, tachypneic with subcostal
recession. Respiratory rate is 70/min. Temperature is 37.8oC. There is some
cyanosis, examination of the chest reveals a bilateral ronchi best heard at the end
of expiration and a few crepitations.
a) What is the most likely diagnosis?
Answer: Bronchiolitis

b) List 3 likely causative organisms


Answer:
➢ Respiratory syncytial virus
➢ Parainfluenza virus
➢ Adenovirus

c) Describe in one or two sentences the pathophysiological process of this


condition
Answer: Viral infection causes inflammatory obstruction (edema and mucus)
of bronchioles. There may be bronchiolar spasms which also further increases
the obstruction. During expiration airways collapse and there is air trapping.
Obstruction affects expiration more than inspiration.

d) What is your management?


MK

Answer:
➢ Admit patient
➢ Provide moist oxygen (keep oxygen saturation more than 92%), continuous
positive airway pressure (CPAP) or assisted ventilation if there is respiratory
failure
➢ Nasal bulb suction
➢ Hydration with IV fluids
➢ Position at 30-40o
➢ Nebulized bronchodilators (albuterol and ipratropium) and steroids may help
some patients
➢ Aerosolized ribavirin if available

60.A 2 days old baby comes into the neonatal unit with history of developing
jaundice since a few hours after birth. On examination, you notice that the child
is weak, hypotonic, deeply jaundiced and pale. He is the 2nd born, the first sibling
did not suffer from this condition.
a) What is the most likely full diagnosis of this illness in the neonate?
Answer: Hemolytic disease of the newborn

b) List 4 investigations that you are going to do in order to confirm the diagnosis
Answer:
➢ Direct Coomb’s test
➢ Full blood count
➢ Peripheral smear
➢ Serum bilirubin

61.An 11 months old infant presents to your admission ward convulsing. There is
history of 2 former such episodes of convulsing. According to the mother, the
convulsions involve the whole body and last about 3 minutes. The child has
temperature 39oC.
a) What is the most likely diagnosis?
Answer: Febrile convulsion

b) List 3 disease condition that would possibly have caused this condition
Answer:
➢ Severe Malaria
➢ Sepsis
MK

➢ Pneumonia

c) Indicate 2 cardinal investigations that would help you manage this child.
Answer:
➢ Malaria parasite slide
➢ Full blood count with differential

62.Joanna a 3-year-old toddler, is brought to AMEU with history of diarrhea and


vomiting for the past 3 days. On examination, she is lethargic, too weak to take
fluid orally and passed scanty urine all day. Her weight is 14kg
a) What plan are you going to apply in rehydrating Joanna?
Answer: WHO PLAN C

b) What fluid are you going to use in treating this little girl?
Answer: Ringer’s lactate or ½ strength Darrows

c) Give details of the plan you are going to use in rehydrating Joanna
Answer:
➢ Give 100ml/kg Ringer’s lactate IV in 3 hours (1400ml in 3 hours)
- Give 420ml in 30 minutes then
- Give 980ml in 2hours 30 minutes
➢ Reassess every 15 minutes if hydration status not improving increase speed
of infusion.
➢ Reassess after 3 hours and re-classify hydration status, if hydrated switch to
appropriate plan.

63.Hamaundu a 4-year-old-boy, is brought to the OPD with edema of both hands


and feet and sores in the mouth. He weighs 8.5kg.
a) Assign the nutritional status to Hamaundu based on welcome classification
Answer:
Expected weight for a 4-year-old child = 2 x (4 + 4) = 16kg
Hamaundu’s weight= 8.5 kg
8.5
W/A percentage= × 100= 53.1% of expected weight
16
Hamaundu has Marasmic Kwashiorkor

b) How many milliliters of F75 would you need to cover his caloric requirement
estimated at 1500kcal/24 hours?
MK

Answer:
Each 100ml of F-75 has 75kCal
If 100ml F-75 gives 75Kcal, volume needed for 1500kcal= (1500 x 100)/75=
2000ml
Therefore, 2000ml per day of F75 is needed

64.Kasongo a 10-year-old boy of Kasisi, presents with fever of 3 weeks duration.


He was treated for malaria a week ago despite a negative malaria thick smear. He
has remarkably lost weight and is complaining of abdominal pain associated with
constipation. The mother says that he loses his memory at times and would be
incoherent in his speech.
a) Write down your differential diagnosis
Answer:
➢ Enteric fever
➢ Viral hepatitis
➢ Abdominal Tuberculosis
➢ Leukemia
➢ Lymphoma

b) What is your most likely diagnosis?


Answer: Enteric fever

c) List investigations you would carry out


Answer:
➢ Full blood count with differential
➢ Peripheral smear
➢ Blood culture
➢ Urine microscopy/culture and sensitivity
➢ Widal antigen test
➢ Hepatitis profile
➢ Liver function tests and enzymes
➢ Abdominal x-ray
➢ Abdominal ultrasound
➢ Urea, electrolytes, creatinine
➢ Bone marrow culture
MK

65.A 7-year-old girl is brought to LMGH with a history of recent bedwetting and
unexplained weight loss. No history of tuberculosis contact. Her rapid test for
HIV is negative
a) What is your differential diagnosis?
Answer:
➢ Type 1 Diabetes mellitus
➢ Urinary tract infection
➢ Diabetes insipidus
➢ Chronic renal

b) Write down 2 more symptoms that may be associated with this condition
Answer:
➢ Polyuria
➢ Polydipsia

c) List down 3 investigations to ascertain your diagnosis


Answer:
➢ Random blood sugar
➢ Fasting blood sugar
➢ Urinalysis (ketones and glucose)

66. Martha, a 4-year-old girl has had a history of bony pains on and off for 2 weeks.
On examination, she is febrile, underweight, pale and in respiratory distress. She
has cervical and axillary lymphadenopathy.
a) What further questions would you ask to help determine the diagnosis?
Answer:
➢ Any recurrent abdominal pain?
➢ Any history of yellowing of eyes?
➢ Any personal or family history of sickle cell disease?
➢ Any joint swelling?
➢ Any recurrent respiratory infections? Pulmonary TB?
➢ Any bleeding?
➢ Any TB contact?
➢ HIV status

b) Give 2 differential diagnoses?


MK

Answer:
➢ Acute leukemia
➢ Acute chest syndrome in Sickle cell disease

c) What investigations would you do to help ascertain your diagnosis?


Answer:
➢ Sickling test
➢ Full blood count with differential
➢ Peripheral smear
➢ Chest X-ray
➢ Lymph node biopsy

67.Mwila is referred from Kasama for a huge spleen. She has had few episodes of
abdominal pain which the mother attributed to worm infestation. On examination
she is underweight with a tinge of jaundice. The night of her admission, she bled
profusely from the nose and required nasal packing.
a) Write down 3 possible diagnoses?
Answer:
➢ Hyperreactive malarial splenomegaly (Tropical splenomegaly syndrome)
➢ Acute leukemia
➢ Viral hepatitis
➢ Splenic sequestration crises in sickle cell patient
b) List down specific investigations to ascertain your diagnosis
Answer:
➢ Blood investigations
- Rapid diagnostic test for malaria
- Peripheral smear to rule out severe malaria, acute leukemia
- Hepatitis viral panel
- Full blood count and ESR
➢ Bone marrow aspirate

68.These are pictures of the same girl in question 47.


MK

a) Write down your findings


Answer:
➢ Webbed neck
➢ Low posterior hair line
➢ Low set ears

a) What is your diagnosis?


Answer: Turner syndrome (monosomy X)

c) Choose the most likely karyotype


(1) XY
(2) XO
(3) XYY
(4) 47, XX, +t (14q21q)

69.A 24-month-old toddler, well nourished, is admitted with acute diarrhea and
severe dehydration. He weighs 9.6kg
a) What is his expected weight?
Answer:
Expected weight= 2 x (age + 4)
Expected weight= 2 x (2 + 4)
Expected weight= 2 x 6
Expected weight= 12kg

b) What is his body weight percentage loss?


Answer:
MK

12−9.6
Body weight percentage loss= × 100= 20%
12

c) Work out the fluid amount required to cover his deficit?


Answer:
Deficit (100ml/kg) = 100 x 9.6
Deficit= 960ml of Ringer lactate (or ½ strength Darrows solution in 5%
dextrose)
288ml to run 30 minutes and 672ml to run in 2 hours 30 minutes

70.You are called to review a newly diagnosed insulin dependent diabetes mellitus
case. The patient is receiving insulin intravenously and his morning urine analysis
is as follows:
SpGr: 1.025, Protein: +, pH: 6.0, Blood: negative, Ketones: ++++, Glucose:
Negative
a) Explain in few words what may have led to the presence of ketones and the
absence of glucose in urine.
Answer: This could probably be caused by a high dosage of insulin causing
hypoglycemia and hunger. The response of the body to hunger leads to
ketogenesis as an alternative source of energy therefore ketones would be
present in urine and glucose absent

b) In your opinion, what would be the blood sugar levels in this patient? Low,
normal, or high?
Answer: The sugar would low

c) How would you manage this patient?


Answer:
a. Check random blood sugar
b. If blood sugar low, give 5ml/kg of 10% dextrose
c. Adjust insulin dosage and monitor glucose 30 minutes to hourly
d. Give intravenous fluids- DNS
e. Monitor for ketones and glucose in urine

71.A 24-month-old girl presents to hospital with history of diarrhea and vomiting
for 4 days and anorexia for 2 weeks. The mother has recently fallen pregnant but
she has recently separated from her husband. On examination the child is
MK

miserable, apathetic and has bilateral pedal edema and dermatosis. Her weight is
7.2kg. Her muscles are wasted.
a) What is the diagnosis?
Answer: Severe acute malnutrition

b) List 5 critical conditions that you must manage in order to save the life of this
child within the first 48 hours
Answer:
➢ Hypoglycemia
➢ Hypothermia
➢ Dehydration
➢ Electrolyte imbalance
➢ Infection

c) What is the expected weight for age for this child?


Answer:
Expected= 2 x (4 + 2)
Expected= 12kg

d) Give differentials that need to be ruled out every time you have such a case.
Answer:
HIV infections
Tuberculosis
Pneumocystis Jirovercii pneumonia
Malaria
Vitamin A deficiency
Severe anemia
Candidiasis
Worm infestation

72.A boy is brought into hospital with difficulties of breathing and swelling of the
legs. The patient indicates that he is unable to sleep supine on the bed as he runs
out of breath. He also indicates that he develops breathlessness after walking only
a few steps. On examination, the boy has a pansystolic murmur in the mitral area
radiating to the left axilla. He also has anemia and fever as well as splenomegaly.
a) Give your differential diagnosis
Answer:
MK

a. Biventricular heart failure secondary to acute rheumatic heart disease


b. Biventricular heart failure secondary to acute mitral regurgitation New
York heart association class III
c. Nephrotic syndrome
d. Infectious mononucleosis

b) Line up investigations based on your differential diagnosis


Answer:
a. Anti-streptolysin O antibody titers for rheumatic heart disease
b. Throat swabs for culture
c. Chest X-ray may show cardiomegaly, pulmonary edema
d. ECG and ECHO for the heart failure
e. Urinalysis for proteinuria

c) Which organisms are usually associated with this condition?


Answer: Lancefield Group A Beta hemolytic streptococcus

d) Which one is the most affected age group and why?


Answer: children between 5-15 years of age are most affected because this
group is most susceptible to streptococcal throat infections with M serotypes
of group A beta hemolytic streptococcus.

e) How would you manage this child?


Answer:
a. Admit patient
b. Keep child propped up
c. Administer humidified oxygen
d. Sedation with morphine sulfate 0.1 to 0.2 mg/kg/dose SC every 4 hours
as needed or Phenobarbital 2 to 3mg/kg/dose PO or IM
e. Give furosemide 1-2mg/kg/day IV in 2-3 divided doses
f. Give digoxin and captopril/enalapril according to age of the child
g. Monitor input and output chart, daily weighing and salt and fluid
restriction
h. Aspirin 100mg/day in 4 to 6 divided for 2-4 months with Prednisolone
2mg/kg/day in 4 divided doses
i. Cover on antibiotics Benzathine penicillin IM one dose followed by
oral penicillin for 10 days
MK

73.A 13-year-old girl is brought into the emergency room with history of difficulty
breathing since the previous night. She is a known sickle cell disease patient who
was lost to follow up. On examination the girl is in severe respiratory distress
with a respiratory rate of 70/min, with sub-costal and intercostal recession. Her
temperature is 38.7oC. Her hemoglobin is 7g/dl and her oxygen saturation using
a pulse oximeter is 76.
a) What are the 2 key differential diagnoses for the condition that the patient is
presenting with?
Answer:
a. Acute chest syndrome
b. Congestive heart failure

b) List 3 investigations you would do in relation with this condition.


Answer:
a. Chest X-ray
b. Full blood count with differential count
c. Blood culture

c) How would you manage this patient?


Answer:
a. Admission
b. Hydration: IV fluids (1.5 x maintenance)
c. Oxygen support- ventilation (CPAP may be necessary)
d. Antibiotics
i. Ceftriaxone and Erythromycin-250mg BD for 10 days
e. Transfuse packed cells (15-20 ml/kg) slowly over 4 hours and give
1mg/kg Furosemide stat for anemia
f. Consider partial exchange transfusion

74.A3 and a half year old female child presents with noisy breathing and fever of 6
hours duration. Prior to this she had been perfectly well. She was born at LMGH
at full term with a birth weight of 3.9kg. There were no neonatal problems. Her
routine vaccinations are up to date and weight gain has been adequate. There is
no family history of note. On examination the patient looked very ill with
temperature 39.7oC, pulse 110/min, BP 110/78, dribbling of saliva from one
MK

corner of the mouth, intercostal and subcostal indrawing with a clear chest. Blood
investigations reveal
Hb- 12g/dl
WBC- 20 x 109/L (polymorphs-84% and lymphocytes-12%)
Sodium 135mmol/L
Potassium 3.5 mmol/L
Urea 4.6 mmol/L

Urinalysis was normal


a) What is the most likely diagnosis?
Answer: Acute epiglottitis

b) What is the most likely causative organism?


Answer: Hemophilus influenza type B

c) Name 3 management measures you are going to institute in this child


Answer:
a. Airway and breathing management
ii. Controlled nasotracheal intubation (consult ENT surgeon)
iii. Humidified oxygen by hood
iv. Keep child in sitting position
v. Avoid examination of the throat
b. Steroids: Dexamethasone 0.3-0.5mg/kg repeat 8 hourly for 2-3 days
max
c. Antibiotics (Third generation cephalosporins): Cefotaxime 100-
150mg/kg/day IM in 2 injections for 5 days

75.Mutinta a 10-year-old girl presents to hospital with history of cough and swelling
of legs for 4 days. On examination she is dyspneic at rest, has bilateral pedal
edema and hepatomegaly. Examination of the heart reveals a pan-systolic
murmur, radiating to the axilla and mid-diastolic murmur in the mitral area. She
also has a diastolic murmur in the aortic area.
a) What is the full diagnosis of this child’s condition?
Answer: Biventricular heart failure secondary to mitral regurgitation with
aortic regurgitation New York heart classification class IV

b) List the valve structural problems that this child has


MK

Answer:
Mitral regurgitation
Aortic regurgitation

c) Indicate the four measures you will take in order to manage the acute phase
of this child
Answer:
a. Administer humidified oxygen
b. Sedation with morphine sulfate 0.1 to 0.2 mg/kg/dose SC every 4 hours
as needed or Phenobarbital 2 to 3mg/kg/dose PO or IM
c. Give furosemide 1-2mg/kg/day IV in 2-3 divided doses
d. Give digoxin

d) What long term treatment would you institute in order to prevent further
deterioration of this child’s condition?
Answer: Continuous monthly prophylaxis with injections of benzathine
penicillin 1,200, 000 IU or Pen V 250mg BD PO can be given. For those
allergic to penicillins erythromycin 250mg BD PO is given until the age of 18
or 21.

76.Nalishebo is a 3-year-old girl who presents with edema and is underweight. The
following are her lab results:
Urea 2.5 mmol/L
Sodium 128 mmol/L
Potassium 3.5 mmol/L
Serum albumin 28g/L
Serum cholesterol 2mmol/L
Urinalysis:
Protein 2+
Blood 1+
Leucocytes: Numerous
a) What 2 other important physical features should be present in this child?
Answer:

b) What is the most likely diagnosis?


Answer: Nephrotic syndrome
MK

c) What other diagnoses should be considered and treated?

77.A 10-year-old boy, Judge Siachinji, whose parents just moved to live in Lusaka
from Siavonga presented at LMGH with terminal hematuria
a) What is the likely diagnosis for this boy?
b) What investigations would you carry out to confirm the diagnosis in this boy
and what would be the finding consistent with the diagnosis?
c) Write down 2 complications from this condition that the boy may develop if
left untreated?
d) How would you treat this child?

78.Zambia ranks one of the countries in Sub-Saharan Africa with the worst infant
and childhood mortality rates. One of the ways to avert this problem is to
reinforce the aims and objectives of the under-five clinics
A. State the major aims of under-five clinics in Zambia as captured on the under-
five card?
B. List the preventable disease from which a Zambian child is immunized on the
current EPI?

79.An 8-year-old girl presented at LMGH with restlessness, heart palpitations and
was rather dazed in general appearance. Investigations showed an impaired
glucose tolerance test with:
Total serum thyroxin 186nmol/L (normal range 10-30)
Free T4 12.5pmol/L (normal range 9.4-24.5)
Free T3 11.8pmol/L (normal range 2.9-8.9)
Serum TSH <0.1Mu/L (normal range 0.7-2.5)

a) What is the most likely clinical diagnosis for this patient?


b) What is the pathophysiological basis of the heart palpitations and restlessness
in this child?
c) Write down 2 other (physical findings) consistent with this condition you
would likely elicit on physical examination of this child?
MK

80.A 9-year-old-boy is referred to LMGH from a mission Hospital for further


investigation. He had developed progressive abdominal distension over a period
of 7 months, and had reduced exercise tolerance compared to his peers.
Examination revealed a frightened but co-operative child who could only
understand Lozi. He was pale, anicteric and weighed 38 kg. Congestion of the
jugular veins was evident on deep inspiration. The pulse varied between 90-97
beats per minute and the heart sounds were normal. Auscultation of the chest
revealed a few coarse crepitations in the bases. The abdomen was uniformly
distended and invariably impossible to palpate for organomegaly. Shifting
dullness was easy to elicit and there was pitting oedema above both ankles.
a) What is your provisional diagnosis?
b) What 3 investigations would you ask for to support this diagnosis?
c) How would you mange this child?

81.List 10 parameters signifying severe malaria


Answer:
A. Convulsions (>2 generalized tonic clonic) other than febrile convulsion or
Altered level of consciousness (cerebral malaria)
B. Cardiovascular collapse (algid malaria)
C. Pulmonary oedema
D. Respiratory Distress syndrome
E. Severe anemia
F. Black water fever
G. Acute renal failure
H. Metabolic acidosis
I. Hypoglycemia
J. Hyperparasetemia

82.List the 10 steps in the management of severe acute malnutrition


Answer:
1. Treat and prevent hypoglycemia
2. Treat and prevent hypothermia
3. Treat and prevent dehydration
4. Treat and prevent infections
5. Correct electrolyte imbalances
6. Give micronutrients
MK

7. Initiating F-75 feeds


8. Catch up F-100 feeds
9. Sensory stimulation and tender loving care
10.Discharge and follow up
11.
12.
MK

SECTION 4: OSCE STATIONS


1. A 10-year-old presents to the hospital with generalized body swelling (anasarca) and an
elevated temperature of 38oC. Preliminary investigations are done. A full blood taken shows
Hemoglobin 8g/dl
White blood cell count 8 x 109/L
Neutrophils 30%
Lymphocytes 20%
Platelets 33 x109/L
Red cell distribution width- 26%

A blood slide is collected and results are shown in the image below

A. What abnormalities are present from the full blood count?


Answer:
• Low hemoglobin
• Low platelet count
• Increased red cell distribution width
• Neutropenia

B. What does the blood slide show?


Answer: Ring forms (Plasmodium falciparum)

C. What is the full diagnosis?


Answer: Severe malaria (Acute renal failure)

D. What other investigations would you undertake?


Answer:
MK

• Coagulation studies: Prothrombin time, activate partial thromboplastin time, clotting


time
• Serum glucose
• Serum urea, creatinine and electrolytes
• Liver function tests

E. How would you manage this child?


Answer:
• Admit child to ICU
• Assess level of consciousness and vitals
• Administer oxygen
• Catheterize and cannulate
• Intravenous artesunate 2.4 mg/kg IV at 0, 12 and 24 hours and if child improves and
can take oral medication (artemether and lumefantrine): 3 tablets twice a day for 3 days.
• Antibiotics: 3rd generation cephalosporins and aminoglycosides or beta lactam+ beta
lactamase inhibitor combination)
• Administer Fresh frozen plasma and intravenous vitamin K
• Furosemide 1mg/kg (maximum 5mg/kg)
• If hypoglycemic give 5% of 10% dextrose IV followed by infusion of 10%.
• Acetaminophen to induce defervescence
• Monitor urine output and metabolic/coagulation panel (electrolytes, urea, creatinine
and liver function tests as well as PT/INR, PTT)

F. What are the other complications associated with this condition?


Answer:
• Cerebral malaria: >2 generalized tonic clonic convulsion and/or impaired level of
consciousness/ unarousable coma attributed to malaria
• Acute non-cardiogenic pulmonary edema
• Acute respiratory distress
• Cardiovascular collapse (shock)-Algid malaria
• Severe anemia Hb <6g/dl
• Hypoglycemia <2.2mmol/dl
• Metabolic acidosis pH<7.25 or bicarbonate <15mEq/L
• Blackwater fever (hemoglobinuria)
• Disseminated intravascular coagulation and abnormal spontaneous bleeding
• Hyperparasitemia
MK

2. A child presents to ED with fever, headache. A chest X-ray is taken and is shown below.

a) Describe the X-ray


Answer: Diffuse bilateral miliary shadowing (miliary picture)

b) Give 5 differentials
Answer:
➢ Miliary Tuberculosis
➢ Histoplasmosis
➢ Coccidiomycosis
➢ Sarcoidosis
➢ Blastomycosis
➢ Cryptococcosis

c) What is the most likely diagnosis?


Answer: Miliary Tuberculosis (Disseminated Tuberculosis) with meningeal involvement

d) What are the complications of your diagnosis?


Answer:
• Meningeal involvement (Tuberculous meningitis)
• Intracranial tuberculomas
MK

e) What investigations would you order?


Answer:
➢ Full blood count with differential count
➢ ESR and C-reactive protein
➢ Gastric lavage/sputum for AFB and GeneXpert
➢ Urine LAM
➢ Retroviral test
➢ CT scan of the head

f) How would you manage?


Answer:
2 months of
➢ Isoniazide 10mg/kg
➢ Rifampicin 15mg/kg
➢ Ethambutol 20mg/kg
➢ Pyrazinamide 35mg/kg
10 months of Isoniazide, rifampicin and supplementation with Vitamin B6 (pyridoxine)

3. Below is a picture of a child that presents to the emergency department with respiratory distress
and stridor

a) Describe the abnormalities shown in the picture


Answer:
➢ Pseudomembrane seen in posterior oropharynx
➢ Bull neck

b) What is the diagnosis?


Answer: Diphtheria

c) What is the causative organism?


Answer: Corynebacterium diphtheriae
MK

d) What is the type of inflammation?


Answer: Acute pseudomembranous inflammation

e) What is your differential diagnosis?


Answer:
➢ Acute membranous tonsillitis
➢ Aphthous stomatitis
➢ Thrush
➢ Infectious mononucleosis

f) What are the complications of this condition?


Answer:
Neurological:
➢ Cranial nerve palsies
➢ Palatal palsy
➢ Generalized polyneuritis
➢ Muscular paralysis (descending)
Respiratory
➢ Respiratory failure
Cardiovascular
➢ Myocarditis
Renal
➢ Proteinuria
➢ Renal tubular necrosis

g) How can this condition be prevented?


Answer:
➢ Diphtheria, pertussis, tetanus-hepatitis B- Hemophilus influenza type B vaccine for
unvaccinated children given at 6 weeks, 10 weeks and 14 weeks.
➢ Give all vaccinated house hold contacts a diphtheria toxoid booster
➢ Give all vaccinated household contacts benzathine penicillin then give them diphtheria
toxoid
o 600, 000 U for <5 years
o 1,200,000 U for >5 years

h) What investigations would you carry out?


Answer:
➢ Throat/oropharynx and larynx swabs for microscopy (Albert stain), culture (takes 8
hours) and sensitivity
➢ Blood cultures
➢ Full blood count with differential
➢ Urinalysis
MK

➢ Erythrocyte sedimentation rate


➢ Schick test-this is a skin test used to determine whether or not a person is susceptible
to diphtheriae
➢ C-reactive protein

i) How do you treat this child?


Answer:
➢ Bed rest for 2 to 3 weeks
➢ Monitor airway as tracheostomy may be required
➢ Administer oxygen
➢ Gain venous access and run fluids
➢ Antitoxin- 40,000 U IM/IV (repeated doses of antitoxin may be given if clinical
improvement is suboptimal)
➢ Procaine benzyl penicillin 50mg/kg for 10 days
➢ Erythromycin can be used as an alternative for those allergic to penicillins.

4. Look at this picture of a child admitted two weeks ago with generalized body swelling

a) What is your differential diagnosis?


Answer:
➢ Nephrotic syndrome
➢ Severe acute malnutrition
➢ Biventricular heart failure
➢ Chronic kidney failure
➢ Liver failure
➢ Protein losing enteropathy
MK

b) Urinalysis was done and results were as follows: Ketones 1+, Glucose nil, Proteins 4+, and
Blood 1+, what is your diagnosis?
Answer: Nephrotic syndrome

c) What investigations would you carry out for your differential diagnosis?
Answer:
➢ 24-hour urine analysis for proteins in urine (for nephrotic syndrome)
➢ Protein-creatinine ratio or albumin-creatinine ratio
➢ Urine microscopy, culture and sensitivity
➢ Urea, creatinine and electrolytes for kidney failure
➢ Liver function tests: serum albumin (may be low in SAM, nephrotic and protein losing
enteropathy), prothrombin time, and serum bilirubin
➢ Liver enzymes may be raised in liver failure
➢ Blood: lipid profile, FBC, C3 and C4 assay
➢ ECG, CXR, ECHO
➢ Abdominal ultrasound

d) Which drugs will you use (including dosage) in this condition and list five side effects?
Answer:
➢ Prednisolone 2mg/kg/day (max dose of 60mg/kg) for 4 to 6 weeks then 1.5mg/kg/day
alternating days for 2 to 5 months (max 40mg/kg).
o Side effects: immunosuppression and recurrent infections, osteoporosis,
lightening and thinning of skin, acne, peptic ulceration, cataracts, easy bruising,
weight gain and abdominal fat distribution on face, neck and trunk, aseptic
necrosis of the femoral head, hypertension, abdominal striae, mood changes
➢ 20% Albumin 0.5g/kg/day with furosemide 1mg/kg/day if indicated in severe ascites
given over 2 hours or spironolactone 2mg/kg
o Side effects of albumin: hypertension
o Furosemide: hypokalemia, hypocalcemia, hypomagnesemia, hyponatremia
o Spironolactone: gynecomastia, skin rash, steven Johnson/TENs.
➢ Gentamicin (antibiotic cover) 7.5mg/kg OD and X-pen 50,000 to 100,000 IU/Kg QID
o Side effects of gentamicin: ototoxicity
o Side effects of X-pen: injection site reactions, diarrhea, stomach upset, nausea and
vomiting, anaphylactic reaction

e) How would you manage the child?


Answer:
➢ Management is supportive
➢ Hospitalize and monitor
o Daily weight
o Urine output
MK

o 24-hour urine protein


o Blood pressure
➢ Diet
o Restrict salt intake
o Restrict fluids
o Monitor protein intake
➢ Avoid infection: treat infection e.g. UTIs
➢ Give influenza vaccine
➢ Avoid thrombosis: Low dose aspirin
➢ For edema infuse salt free albumin 0.5-1g/kg infused over 4 hours followed by
furosemide 1-2 mg/kg IV
➢ Curative: Prednisolone 2mg/kg/day (max dose of 60mg/kg) for 4 to 6 weeks then
1.5mg/kg/day alternating days for 2 to 5 months (max 40mg/kg).

5. Below are hemogram findings taken from a patient admitted on the ward:
Hemoglobin 3g/dl,
Platelets 160, 000/mm3
RDW-15%,
MCH-20

a) What is the diagnosis?


Answer: Severe microcytic anemia

b) List 6 causes
Answer:
➢ Iron deficiency anemia
➢ Anemia of chronic disease
➢ Thalassemia
➢ Sideroblastic anemia
➢ Chronic hemorrhage e.g. peptic ulcer disease
➢ Hookworm infestation
➢ Lead poisoning

c) List 6 investigations?
Answer:
➢ Blood
o Sickling test
MK

o Coombs test
o Peripheral smear
o Iron studies: Serum iron, Ferritin, Serum transferrin and total iron binding capacity
➢ Stool
o Stool for occult blood
o Stool Microscopy/culture/sensitivity for ova and parasites

d) How do you treat?


Answer:
➢ Treatment is dependent on cause
o Iron deficiency: treat any underlying infection, ferrous sulfate 200mg TDS
o Hookworm infestation: deworm with mebendazole
o Sideroblastic anemia: Vitamin B6 supplementation and blood transfusions
o Anemia of chronic disease: treat the underlying disease condition, Erythropoietin
can be given
o Thalassemia: blood transfusion
o Lead poisoning: use chelating agents
➢ For symptomatic patient: blood transfusion

6. A 13-year-old female is brought unconscious and hyperventilating.


a) What important questions would you ask on your history?
Answer:
➢ Child’s health and activity in the last 24 hours
➢ History of similar illness
➢ Pre-existing neurological disability
➢ History of weight loss, weakness, fatigue, excessive urination, excessive hunger and
eating habits, recurrent oral or vaginal thrush, abdominal pain, vomiting and fever
➢ Personal and family history of diabetes and asthma
➢ Drug history (name and dosage)
➢ History of ingesting any poisoning or noxious substances
➢ Last meal
➢ History of drinking alcohol or substance abuse
➢ History of trauma

b) What are the possible causes?


Answer:
➢ Diabetic ketoacidosis
➢ Hypoglycemia
➢ Asthma
➢ Salicylate poisoning
➢ Organophosphate poisoning
➢ Paracetamol poisoning
MK

c) How would you manage?


Answer:
➢ Admit
➢ ABCs
o Ensure airway is patent and suction any secretions
o Ensue patient is breathing and administer oxygen
o Gain venous access and draw blood for FBC, Urea, electrolytes, creatinine, blood
sugar, liver function tests and arterial blood gasses
o Nurse in a propped up position
o Perform neurological examination- assess GCS/AVPU, Light pupillary reflex, and
level of dehydration
➢ Fluids= 100ml/kg ringer’s lactate (deficit) + maintenance to run over 48 hours (when
sugar drops below 13 mmol/L change to DNS)
➢ Add 20mmol of potassium chloride to second bag of fluids
➢ Catheterize patient and monitor urine output
➢ Insulin: 0.1IU/kg in 6 hours as an infusion in 60ml of Normal saline
➢ Continuous monitoring of
o Vitals and blood glucose hourly
o ECG
o Electrolytes, urea and creatinine
➢ Cover on antibiotics: X-pen 50, 000- 100,000 IU/kg QID and gentamicin 7.5mg/kg
OD
d) What investigation would you order?
Answer:
➢ Blood:
o Full blood count with differential count
o Random blood sugar
o Urea, electrolytes and creatinine
o Rapid diagnostic test and malaria parasite slide
o Arterial blood gasses
o Liver function tests
➢ Urinalysis

e) What are the complications?


Answer:
➢ Cerebral edema
➢ Pulmonary edema
➢ Electrolyte imbalance e.g. hypokalemia
➢ Kidney injury and renal failure
MK

7. A child presents with a purulent cough and finger clubbing as shown in the picture below. An
HIV test is done and it is negative.

a) What is your differential diagnosis?


Answer:
➢ Bronchiectasis
➢ Lung abscess
➢ Bronchogenic carcinoma

b) What is the most likely diagnosis?


Answer: bronchiectasis

c) What will you look for clinically?


Answer:
➢ History:
o Hemoptysis
o Foul smelling sputum
o Weight loss
➢ Examination:
o Finger clubbing
o Tachypnea
o Wheezing and rhonchi
o Crackles
o Features of right sided heart failure (peripheral edema)- cor- pulmonale

d) What investigations will you carry out


Answer
➢ Sputum for microscopy, culture and sensitivity
➢ Sputum for GeneXpert
➢ Rheumatoid factor
➢ Full blood count (to rule out eosinophilia in allergic aspergillosis)
MK

➢ Chest X-ray: hyperinflation (flattened diaphragm), scattered irregular opacities, honey


comb appearance
➢ CT chest: dilated walls and thickness

e) How would you management this child?


Answer:
➢ Non pharmacological: postural drainage
➢ Pharmacological:
o Treatment is dependent on cause
o Bronchodilators
o Mucolytics

8. A 3-year-old presents to the emergency department wheezing.


a) What investigations are you going to order?
Answer:
➢ Full blood count
➢ Chest X-ray
➢ Arterial blood gasses
➢ Allergy screening

b) How would you management? (List classes of drugs used in the management including
their mechanisms of action for each)
Answer:
➢ Admit
➢ ABC
o Ensure airway is patent, suction any secretions
o Ensure patient is breathing and administer oxygen via face mask, keep saturation
>92%
o Nebulize bronchodilators
o Beta 2 agonists (Salbutamol 2.5mg): stimulates beta 2 receptors on bronchial
smooth muscle and through a G-stimulatory cAMP cascade results in
phosphorylation of myosin light chain kinase (MLCK) and inhibition of
MLCK leading to relaxation of smooth muscle and bronchodilation.
o Antimuscarinic agents e.g. Ipratropium bromide 250mcg 4-6 hourly: this is
an antimuscarinic agents that blocks M3 receptors associated with
parasympathetic stimulation of bronchial air passageways
o Methylxanthines (Aminophylline)- this is a phosphodiesterase inhibitor that
inhibits the breakdown of cAMP to AMP, thereby maintaining the
concentration of cAMP in the cell which causes bronchodilation
o Steroids e.g. Prednisolone 1-2mg/kg (40mg max) for 3 days PO: This is a steroid,
it inhibits phospholipase A, preventing the conversion of phospholipids into
arachidonic acid thereby inhibit the further production of leukotrienes and
MK

prostaglandins (chemical mediators of inflammation) in both the lipo-oxygenase


and cyclo-oxygenase pathways. It acts as an anti-inflammatory agent.
o Magnesium sulfate- reduces inflammation, as well as acts as a bronchodilator.
➢ Monitor Vitals

9. Below is a picture of a brain CT of a patient being followed up at LMGH.

a) What are the abnormalities shown in the CT image?


Answer: Dilatation of both lateral ventricles

b) What is the most likely diagnosis?


Answer: Hydrocephalus

c) Classify the causes and give one pathology in each


Answer:
➢ Congenital:
o Stenosis of the cerebral aqueduct
o Dandy walker malformation: absent or hypoplastic cerebellar vermis and cystic
enlargement of 4th ventricle that blocks flow of CSF (posterior fossa cyst
continuous with 4th ventricle)
o Arnold chiari malformation: herniation of cerebellum and medulla through
foramen magnum
➢ Acquired
o Infectious: TB meningitis, chronic and pyogenic meningitis
o Non-infectious: Posterior fossa tumors (medulloblastoma, astrocytoma,
ependymoma), intracranial hemorrhage, rupture aneurysm

d) What is your differential diagnosis?


Answer:
➢ Achondroplasia
➢ Familial macrocephaly
➢ Cretinism
MK

e) What is the management?


Answer:
Medical:
➢ Daily measurement of occipital frontal circumference
➢ Acetazolamide 25 to 100 mg/kg/day (carbonic anhydrase inhibitor)
➢ Treat meningitis
Surgical (shunts)
➢ Ventriculoperitoneal
➢ Ventricular-atrial
➢ Ventricular-osseous
➢ Ventricular spinal
➢ Ventricular-pleural

f) Complications of management
Answer:
➢ Acute:
o Infection
o Slit ventricular syndrome (small ventricles formed in shunt)
➢ Long term
o Blockage of shunt
o Dislodging of shunt
o Shortening of shunt relative to growth of child
o Shunt nephritis

10. Concerning unconsciousness, how would you assess the coma state with parameters.
Answer:
➢ AVPU (quick assessment)
o A-alert
o V- responding to verbal stimuli
o P- responding to pain
o U- unresponsive

➢ Blantyre coma scale (Children <5 years)- minimum score 0, maximum score 5
o Eye movement
1- Watches or follows (focusing)
0- Fails to watch or follow (none)

o Best verbal response


2- Cries appropriately with pain or if verbal, speaks (Cries appropriately)
1- Moan or abnormal cry with pain (inappropriate cry)
0- No vocal response to pain (none)
MK

o Best motor response


2- Localizes painful stimulus (patient’s ability to remove stimuli)
1- Withdraws limb from painful stimulus
0- No response

➢ Glasgow coma scale (children >5 years): Minimum score- 3, maximum score-15
o Eye opening
4- Spontaneous
3- To verbal stimuli
2- To pain
1- None

o Best verbal response


5- Oriented
4- Confused
3- Inappropriate words
2- Incomprehensible sounds
1- None

o Best motor response


6- Obeys verbal command
5- Localizes pain
4- Withdraws from pain
3- Flexion to pain (decorticate)
2- Extension to pain (decerebrate)
1- None

11. A patient presents with 2 weeks polyuria.


a) What are the causes?
Answer:
➢ Diabetes mellitus
➢ Diabetes insipidus
➢ Urinary tract infection

b) How would you investigate?


Answer:
➢ Blood
o Random blood sugar
o Fasting blood sugar
o Urea, electrolytes and creatinine
o Serum ADH
o Full blood count with differential
➢ Urinalysis
MK

c) What is the most likely diagnosis?


Answer: Diabetes mellitus

d) What is the management?


Answer:
➢ Admit
➢ Manage diabetic ketoacidosis
o ABCs
o Fluids= 100ml/kg ringer’s lactate (deficit) + maintenance to run over 48 hours
(when sugar drops below 13 mmol/L change to DNS)
o Add 20mmol of potassium chloride to second bag of fluids
o Catheterize patient and monitor urine output
o Insulin: 0.1IU/kg in 6 hours as an infusion in 60ml of Normal saline
o Continuous monitoring of Vitals and blood glucose hourly, ECG Electrolytes, urea
and creatinine
o Cover on antibiotics: X-pen 50, 000- 100,000 IU/kg QID and gentamicin 7.5mg/kg
OD
➢ Counsel on Diabetes mellitus clinical features, complication and treatment plan.
➢ Institute insulin regimen on discharge

12. A patient present with history of fever, headache, and neck pain. Histology shows the picture
shown below

a) What is the diagnosis?


Answer: Bacterial meningitis

b) What is the causative organism?


Answer: Neisseria meningitidis

c) What investigations would you order?


Answer:
MK

➢ Full blood count with differential for infections


➢ Lumbar puncture for CSF biochemistry, microscopy, culture and sensitivity
➢ Blood culture and sensitivity

d) How would you treat?


Answer:
➢ Admit patient
➢ Check blood glucose and treat hypoglycemia (5ml/kg of 10% dextrose)
➢ Antibiotics: IV cefotaxime 100mg/kg QID for 10-14 days or Crystalline penicillin 100,000
IU/Kg QID
➢ Monitor vitals, pupillary reflexes and level of consciousness 4 hourly
➢ Fit chart
➢ IV fluids- 2/3 of fluid requirement
➢ Antipyretic: paracetamol IV
➢ Treat seizures if present: diazepam 0.3 mg/kg/dose IV or 0.5mg/kg/dose rectal

e) What are the complications?


Answer:
➢ CNS complications
o Subdural effusion or empyema
o Brain abscess
o Hydrocephalus
o Long term neurological deficits
- Neurosensory deafness
- Learning disability
- Nerve palsies
➢ Systemic complications
o Shock
o Syndrome of inappropriate ADH secretion
o Status epilepticus

13. A baby born at 28 weeks is brought to the NICU.


MK

a) What questions would you ask in the history?


Answer:
➢ Risk factors for prematurity
o Maternal age
o Multiple pregnancy
o Infection, maternal illness e.g. pregnancy induced hypertension, cervical
incompetence, antepartum hemorrhage
➢ Obstetric history
➢ Condition at birth: APGAR score, resuscitation required
➢ Birthweight
➢ Gestational age
➢ Any antenatal steroid given to mother

b) What are the risk factors for preterm birth?


Answer:
➢ Maternal factors:
o Pre-eclampsia
o Infections especially group B streptococcus
o Chronic medical illness e.g. heart or kidney disease, sickle cell disease
o Previous preterm delivery
➢ Uterine/placental factors:
o Placenta previa
o Placenta abruption
o Premature rupture of membranes
o Polyhydramnios
o Cervical incompetence
➢ Fetal causes
o Multiple gestations
o Congenital anomalies
MK

c) What investigations would you do?


Answer
➢ Full blood count with differential count
➢ Lecithin-sphingomyelin ratio
➢ Urea, creatinine and electrolytes
➢ Liver function tests
➢ Blood culture
➢ Urinalysis
➢ Urine microscopy/culture/sensitivity
➢ Thyroid function

d) How would you manage this child?


Answer:
➢ Admit to neonatal intensive care unit
➢ Temperature control using an incubator
➢ Nutrition, feeding and intravenous fluid: total parental nutrition, delay enteral feeds and if
given use expressed breast milk
➢ Cover on antibiotics (cefotaxime and cloxacillin)
➢ Vitamin K 1mg IM
➢ Treat complications

e) What are the complications of this condition?


Answer:
CNS
➢ Periventricular and Intraventricular hemorrhage
➢ Cerebral palsy
➢ Learning disability
➢ Deafness
➢ Retinopathy of prematurity
➢ Blindness

Respiratory system
➢ Respiratory distress syndrome (hyaline membrane disease)
➢ Apnea of prematurity
➢ Chronic lung disease/bronchopulmonary dysplasia

Cardiovascular
➢ Persistent/patent ductus arteriosus
➢ Atrial septal defects
➢ Ventricular septal defects
➢ Anemia of prematurity
MK

Gastrointestinal system
➢ Necrotizing enterocolitis
➢ Umbilical hernia

Metabolic system
➢ Hypoglycemia
➢ Hypothermia
➢ Osteopenia of prematurity

14. A 5-month-old presents with fever and convulsions for 20 minutes


a) What questions would you ask in the history?

b) What is your differential diagnosis?


Answer:
➢ Status epilepticus
➢ Bacterial meningitis
➢ Severe malaria
➢ Sepsis

c) What investigations would you carry out?


Answer:

d) How would you manage?

15. Picture of X-ray of lower limb with tibial expansion/swelling


a) What is your differential diagnosis?
Answer:
➢ Osteomyelitis
➢ Cellulitis
➢ Pyomyositis
➢ Fasciitis
➢ Trauma
➢ Bone infarction VOC in SCD
MK

➢ Osteosarcoma

b) What organisms cause this?


Answer:
➢ <1 year: Group B streptococcus, Staphylococcus aureus and E. coli
➢ 1 year-16 years: Staphylococcus aureus, Streptococcus pyogenes and Hemophilus
influenza
➢ >16 years: Staphylococcus aureus, Staphylococcus epidermidis and Gram negative
bacteria
➢ In sickle cell disease: Salmonella

c) What investigations would you carry out?


Answer:
➢ Full blood count with differential
➢ Blood culture
➢ ESR and C-reactive protein

d) How would you manage?


Answer:
➢ Medical
- Antibiotics:
o Chloramphenicol 25mg/kg TDS
o Cloxacillin 50mg/kg IM or IV QID
o If suspecting salmonella: Ciprofloxacin
- Analgesics for pain management
➢ Surgical: sequestrectomy or if acute bone drilling

16. Patient presents with hematuria and a CXR showing a left upper lobe opacity with cavitation
17. What is your differential diagnosis?
Answer:
➢ Disseminated TB
➢ Pulmonary bacterial abscess
➢ Bronchogenic carcinoma

18. What is your most likely diagnosis?


Answer: Disseminated TB
MK

19. How would your explain the blood in urine?


Answer: hematogenous dissemination of TB to the kidneys

20. What investigations would you carry out?


Answer:
➢ Full blood count
➢ ESR and C-reactive protein
➢ Gastric lavage/sputum for AFB and GeneXpert
➢ Urine LAM
➢ Retroviral test

21. What advice would you give this patient’s caregivers?


Answer:
➢ TB treat will be 12 months instead of 6 months so advise will be on strict adherence
to medication and monitoring of the child for new symptoms and side effects.

22. How would you treat this patient?


Answer:
2 months of
➢ Isoniazide 10mg/kg
➢ Rifampicin 15mg/kg
➢ Ethambutol 20mg/kg
➢ Pyrazinamide 35mg/kg
10 months of Isoniazide, rifampicin and supplementation with Vitamin B6 (pyridoxine)

23. CXR with a globular heart and horizontalization of the ribs.


a) What are the X-ray features?
b) List 3 differential diagnosis
c) What features will you hear on auscultation?
d) What are the complications
MK

24. Pelvic X-ray with osteoporotic features, patient has limp. Hemogram hb8
a) Describe the X-ray
b) What investigations would you order
c) What are the long term complications
d) What is the management of this condition

25. A child is found chewing paracetamol, unconscious, GCS 8


a) What important questions would you ask in the history?
b) What investigations would you order?
c) How do you assess with the Blantyre coma scale
d) How would you mange
e) List 3 complications

26. Domestic dispute, doom was drunk, brought in unconscious


a) What questions would you ask in the history?
b) What investigations would you order for?
c) How would you manage?

27. Epistaxis, hemogram WBC high, Hb low, Plt Low


a) Give your differentials
b) Patient developed rash on the trunk
1. What is your full diagnosis?
2. What investigations would you carry out?
3. How would you manage?

28. A 15-year-old male patient is brought in with a history of having bloody diarrhea for a day. He
was found in a room with an empty bottle of an unknown substance. He also vomited and is
lethargic
a) What important question would you ask on your history?
Answer:

b) What is the most likely diagnosis?


Answer: Salicylate poisoning
MK

c) What other features do you expect to find?


Answer:
Nausea, vomiting, diaphoresis, tinnitus, vertigo, hyperventilation, tachycardia, and
hyperactivity, hyperthermia

d) What are the likely complications?


Answer:
➢ Hepatitis
➢ Coagulopathies
➢ Perforation

e) What investigations would you order?


Answer:
Salicylate blood levels
Urinalysis
Blood ketones
U/Es
Arterial blood gasses
Endoscopy

f) How are you going to manage?


Answer:
➢ ABCs
➢ Fluids NS 10-20ml/kg/hr over 1-2 hours. Monitor urine output (2-3ml/kg/hr)
➢ Activated charcoal (1g/kg if <1 year, 25-50g/kg if older)
➢ IV sodium bicarbonate (1mmol/kg) + supplementary potassium 2-5mmol/kg in 3-4
divided doses
➢ Monitor glucose
➢ Vitamin K 10mg IM/IV
MK

SECTION 5: VIVAS
LIST OF CONDITIONS
1. Neonatal Jaundice
2. Severe Malaria in children
3. Meningitis in children
4. Birth Asphyxia
5. Nephrotic syndrome
6. Severe Acute Malnutrition
7. HIV in children
8. Tuberculosis in children
9. Sickle Cell Anemia
10.Congenital Heart Diseases
11.Trisomy 21
12.Acute Diarrhoeal Diseases in children
13.Immunizations in children
14.Prematurity
15.Diabetes Mellitus in children

FOCUS
1. Pathophysiology
2. Case Scenarios
MK

3. Management

You might also like